Buy BOOKS at Discounted Price

Electricity

Class 10th Physics Lakhmir Singh And Manjit Kaur Solution
Very Short Answer Type Questions-pg-5
  1. By what other name is the unit JouleCoulomb called?
  2. Which of the following statements correctly defines a volt? (a) A volt is a joule per…
  3. What do the letters p.d. stand for?
  4. Which device is used to measure p.d.?
  5. What is meant by saying that the electric potential at a point is 1 volt?…
  6. How much work is done when one coulomb charge moves against a potential difference of 1…
  7. What is the SI unit of potential difference?
  8. How much work is done in moving a charge of 2 C across two points having a potential…
  9. What is the unit of electric charge?
  10. Define one coulomb charge.
  11. Fill in the following blanks with suitable words : (a) Potential difference is measured in…
Very Short Answer Type Questions-pg-18
  1. Name the law which relates the current in a conductor to the potential difference across…
  2. Name the unit of electrical resistance and give its symbol.
  3. Name the physical quantity whose unit is ohm.
  4. What is the general name of the substances having infinitely high electrical resistance?…
  5. Keeping the resistance constant, the potential difference applied across the ends of a…
  6. State the factors on which the strength of electric current flowing in a given conductor…
  7. Which has less electrical resistance: a thin wire or a thick wire (of the same length and…
  8. Keeping the potential difference constant, the resistance of a circuit is halved. By how…
  9. A potential difference of 20 volts is applied across the ends of a resistance of 5 ohms.…
  10. A resistance of 20 ohms has a current of 2 amperes flowing in it. What potential…
  11. A current of 5 amperes flows through a wire whose ends are at a potential difference of 3…
  12. Fill in the following blank with a suitable word : Ohms law states a relation between…
Short Answer Type Questions-pg-18
  1. Distinguish between good conductors, resistors and insulators. Name two good conductors,…
  2. Classify the following into good conductors, resistors and insulators: Rubber, Mercury,…
  3. What is Ohms law? Explain how it is used to define the unit of resistance.…
  4. What is meant by the resistance of a conductor? Write the relation between resistance,…
  5. When a 12 V battery is connected across an unknown resistor, there is a current of 2.5 mA…
  6. Define the of resistance (or Define the unit ohm).
  7. What happens to the resistance as the conductor is made thinner?
  8. Keeping the potential difference constant, the resistance of a circuit is doubled. By how…
  9. Why do electricians wear rubber hand gloves while working with electricity ?…
  10. What p.d. is needed to send a current of 6 A through an electrical appliance having a…
  11. An electric circuit consisting of a 0.5 m long nichrome wire XY, an ammeter, a voltmeter,…
Long Answer Type Questions-pg-19
  1. What is the ratio of potential difference and current known as?
  2. The values of potential difference V applied across a resistor and the corresponding…
  3. Name the law which illustrated by the above V-I graph.
  4. Write down the formula which states the relation between potential difference, current and…
  5. The potential difference between the terminals of an electric iron is 240 V and the…
Multiple Choice Questions (mcqs)-pg-19
  1. The p.d. across a 3 resistor is 6 V. The current flowing in the resistor will be:A. B. 1 A…
  2. A car headlight bulb working on a 12 V car battery draws a current of 0.5 A. The…
  3. An electrical appliance has a resistance of 25 . When this electrical appliance is…
  4. When a 4 resistor is connected across the terminals of a 12 V battery, the number of…
  5. Ohms law gives a relationship between:A. current and resistance B. resistance and…
  6. The unit of electrical resistance is :A. ampere B. volt C. coulomb D. ohm…
  7. The substance having infinitely high electrical is called:A. conductor B. resistor C.…
  8. Keeping the potential difference constant, the resistance of a circuit is halved. The…
  9. Keeping the potential difference constant, the resistance of a circuit is halved. The…
Questions Based On High Order Thinking Skills (hots)-pg-20
  1. An electric room heater draws a current of 2.4 A from the 120 V supply line. What current…
  2. Name the electrical property of a material whose symbol is omega.…
  3. The graph between V and I for a conductor is a straight line passing through the…
  4. A p.d. of 10 V is needed to make a current of 0.02 A flow through a wire. What p.d. is…
  5. A current of 200 mA flows through a 4 K resistor. What is the p.d. across the resistor?…
Very Short Answer Type Questions-pg-26
  1. What happens to the resistance as the conductor is made thicker?
  2. If the length of a wire is doubled by taking more of wire, what happens to its resistance?…
  3. On what factors does the resistance of a conductor depend?
  4. Name the material which is the best conductor of electricity.
  5. Which among iron and mercury is a better conductor of electricity?…
  6. Why are copper and aluminium wires usually used for electricity transmission?…
  7. Name the material which is used for making the heating element of an electric iron.…
  8. What is nichrome? State its one use.
  9. Give two reasons why nichrome alloy is used for making the heating elements of electrical…
  10. Why are the coils of electric irons and electric toasters made of an alloy rather than a…
  11. Which has more resistance: (a) a long piece of nichrome wire or a short one? (b) a thick…
  12. How does the resistance of a pure metal change if its temperature decreases?…
  13. How does the presence of impurities in a metal affect its resistance?…
  14. Fill in the following blanks with suitable words : Resistance is measured in .............…
Short Answer Type Questions-pg-26
  1. What do you understand by the resistivity of a substance?
  2. A wire is 1.0 m long, 0.2 mm in diameter and has a resistance of 10. Calculate the…
  3. Write down an expression for the resistance of a metallic wire in terms of the…
  4. What will be the resistance of a metal wire of length 2 metres and area of cross-section…
  5. Give two examples of substances which are good conductors of electricity. Why do you think…
  6. Calculate the resistance of a copper wire 1.0 km long and 0.50 mm diameter if the…
  7. Will current flow more easily through a thick wire or a thin wire of the same material…
  8. How does the resistance of a conductor depend on : (a) length of the conductor? (b) area…
  9. Give one example to show how the resistance depends on the nature of material of the…
  10. Calculate the resistance of an aluminium cable of length 10 km and diameter 2.0 mm if the…
  11. What would be the effect on the resistance of a metal wire of : (a) increasing its length?…
  12. How does the resistance of a wire vary with its : (a) area of cross-section? (b) diameter?…
  13. How does the resistance of a wire change when: (i) its length is tripled? (ii) its…
  14. Calculate the area of cross section of a wire if its length is 1.0 m, its resistance is 23…
Long Answer Type Questions-pg-27
  1. Define resistivity. Write an expression for the resistivity of a substance. Give the…
  2. State the SI unit of resistivity.
  3. Distinguish between resistance and resistivity.
  4. Name two factors on which the resistivity of a substance depends and two factors on which…
  5. The resistance of a metal wire of length 1 m is 26 at 20 C. If the diameter of the wire is…
Multiple Choice Questions (mcqs)-pg-27
  1. The resistance of a wire of length 300 m and cross-section area 1.0 mm^2 made of material…
  2. When the diameter of a wire is doubled, its resistance becomes:A. double B. four times C.…
  3. If the resistance of a certain copper wire is 1, then the resistance of a similar nichrome…
  4. If the diameter of a resistance wire is halved, then its resistance becomes:A. four times…
  5. The resistivity of a certain material is 0.6 m. The material is most likely to be:A. an…
  6. When the area of cross-section of a conductor is doubled, its resistance becomes:A. double…
  7. The resistivity of copper metal depends on only one of the following factors. This factor…
  8. If the area of cross-section of a resistance wire is halved, then its resistance becomes…
Questions Based On High Order Thinking Skills (hots)-pg-27
  1. A piece of wire of resistance 20 is drawn out so that its length is increased to twice its…
  2. The electrical resistivities of three materials P, Q and R are given below : P 2.3 x 10^3…
  3. The electrical resisitivities of four materials A, B, C and D are given below : A) 110 x…
  4. The electrical resistivities of five substances A, B, C, D and E are given below : A) 5.20…
Short Answer Type Questions-pg-5
  1. What is meant by conductors and insulators? Give two examples of conductors and two of…
  2. Which of the following are conductors and which are insulators? Sulphur, Silver, Copper,…
  3. What do you understand by the term electric potential? (or potential) at a point? What is…
  4. State the relation between potential difference, work done and charge moved.…
  5. Calculate the work done in moving a charge of 4 coulombs from a point at 220 volts to…
  6. Name a device that helps to measure that potential difference across a conductor.…
  7. How much energy is transferred by a 12 V power supply to each coulomb of charge which it…
Very Short Answer Type Questions-pg-37
  1. Give the law of combination of resistances in series.
  2. If five resistances, each of value 0.2 ohm, are connected in series, what will be the…
  3. State the law of combination of resistances in parallel.
  4. If 3 resistances of 3 ohm each are connected in parallel, what will be their total…
  5. How should the two resistances of 2 ohms each be connected so as to produce an equivalent…
  6. Two resistances X and Y are connected turn by turn : (i) in parallel, and (ii) in series.…
  7. What possible values of resultant resistance one can get by combining two resistances, one…
  8. Show how you would connect two 4 ohm resistors to produce a combined resistance of (a) 2…
  9. Which of the following resistor arrangement, A or B, has the lower combined resistance?…
  10. A wire that has resistance R is cut into two equal piece. The two parts are joined in…
  11. Calculate the combined resistance in each case:
  12. Find the current in each resistor in the circuit shown below:
Short Answer Type Questions-pg-38
  1. Explain with diagrams what is meant by the series combination and parallel combination of…
  2. A battery of 9 V is connected in series with resistors of 0.2 , 0.3 , 0.4 , 0.5 and 12 .…
  3. An electric bulb of resistance 20 and a resistance wire of 4 are connected in series with…
  4. Three resistors are connected as shown in the diagram. Through the resistor 5 ohm, a…
  5. For the circuit shown in the diagram below: What is the value of: (i) current through 6…
  6. Two resistors, with resistances 5 and 10 respectively are to be connected to a battery of…
  7. The circuit diagram given below shows the combination of three resistors R1, R2 and R3 :…
  8. In the circuit diagram given below, the current flowing across 5 ohm resistor is 1 amp.…
  9. A resistor has a resistance of 176 ohms. How many of these resistors should be connected…
  10. An electric heater which is connected to 220 V supply line has two resistance coils A and…
  11. In the circuit diagram given below five resistances of 10 , 40 , 30 , 20 and 60 are…
  12. In the circuit diagram given below, three resistors, R1, R2 and R3 of 5 , 10 and 30…
  13. A p.d. of 4 V is applied to two resistors of 6 and 2 and 6 connected in parallel.…
  14. A p.d. of 6 V is applied to two resistors of 3 and 6 connected in parallel. Calculate: (a)…
  15. In the circuit shown below, the voltmeter reads 10 V. (a) What is the combined resistance?…
  16. In the circuit given below: (a) What is the combined resistance? (b) What is the p.d.…
  17. A 5 V battery is connected to two 20 resistors which are joined together in series. (a)…
  18. The figure given below shows an electric circuit in which current flows from a 6V battery…
  19. A 4 coil and a 2 coil are connected in parallel. What is their combined resistance? A…
Long Answer Type Questions-pg-41
  1. With the help of a circuit diagram, deduce the equivalent resistance of two resistances…
  2. Two resistances are connected in series as shown in the diagram: (i) What is the current…
  3. With the help of a diagram, derive the formula for the resultant resistance of three…
  4. For the circuit shown in the diagram given below Calculate: (i) the value of current…
  5. With the help of a circuit diagram, obtain the relation for the equivalent resistance of…
  6. In the circuit diagram shown below, find: (i) Total resistance. (ii) Current shown by the…
  7. Explain with the help of a labelled circuit diagram, how you will find the resistance of a…
  8. In the diagram shown below, the cell and the ammeter both have negligible resistance. The…
Multiple Choice Questions (mcqs)-pg-42
  1. The figure given below shows three resistors Their combined resistance is:A. B. 14 C. D.…
  2. If two resistors of 35 and 15 are joined together in series and then placed in parallel…
  3. The diagram below shows part of a circuit: If this arrangement of three resistors was to…
  4. In the circuit shown below : The potential difference across the 3 resistor is:A. B. C. 1…
  5. A battery and three lamps are connected as shown: Which of the following statements about…
  6. V1, V2 and V3 are the p.d. across the 1 , 2 and 3 resistors in the following diagram, and…
  7. A wire of resistance R1 is cut into five equal pieces. These five pieces of wire are then…
Questions Based On High Order Thinking Skills (hots)-pg-43
  1. Show with the help of diagrams, how you would connect three resistors each of resistance 6…
  2. To resistances when connected in parallel give resultant value of 2 ohm; when connected in…
  3. A resistor of 8 ohm. Calculated in parallel with another resistor X. The resultant…
  4. You are given three resistances of 1, 2 and 3 ohms. Show by diagrams, how with the help of…
  5. How will you connect three resistors of 2, 3 and 5 respectively so as to obtain a…
  6. How will you connect three resistors of resistance 2, 3 and 6 to obtain a total resistance…
  7. What is (a) highest, and (b) lowest, resistance which can be obtained by combining four…
  8. What is the resistance between A and B in the figure given below?…
  9. You are given one hundred 1 resistors. What is the smallest and largest resistance you can…
  10. You are supplied with a number of 100 resistors. How could you combine some of these…
  11. The resistors R1, R2, R3 and R4 in the figure given below are all equal in value. What…
  12. Four resistances of 16 ohms each are connected in parallel. Four such combinations are…
  13. If the lamps are both the same in the figure given below and if A1 reads 0.50 A, what do…
Very Short Answer Type Questions-pg-47
  1. Are the lights in your house wired in series?
  2. What happens to the other bulbs in a series circuit if one bulb blows off?…
  3. What happens to the other bulbs in a parallel circuit if one bulb blows off?…
  4. Which type of circuit, series of parallel, is preferred while connecting a large number of…
  5. Draw a circuit diagram to show two 4 V electric lamps can be lit brightly from two 2 V…
Short Answer Type Questions-pg-47
  1. Why is series arrangement not used for connecting domestic electrical appliances in a…
  2. Give three reasons why different electrical appliances in a domestic circuit are connected…
  3. Ten bulbs are connected in a series circuit to a power supply line. Ten identical bulbs…
  4. Consider the circuits given below: (a) In which circuit are the lamps dimmest? (b) In…
  5. If you were going to connect two light bulbs to one battery, would you use a series or a…
Long Answer Type Questions-pg-48
  1. Which is the better way to connect light and other electric appliances in domestic wiring:…
  2. Christmas tree lamps are usually wired in series. What happens if one lamp breaks?…
  3. An electrician has wired a house in such a way that if a lamp gets fused in one room of…
  4. Draw a circuit diagram showing two electric lamps connected in parallel together with a…
Multiple Choice Questions (mcqs)-pg-48
  1. The lamps in a household circuit are connected in parallel because:A. this way they…
  2. Using the circuit given below, state which of the following statement is correct? A. When…
Questions Based On High Order Thinking Skills (hots)-pg-48
  1. Draw a circuit diagram showing two lamps, one cell and a switch connected in series.…
  2. How can you change the brightness of the lamps?
  3. Consider the circuit given below where A, B and C are three identical light bulbs of…
  4. How do you think the brightness of two lamps arranged in parallel compares with the…
  5. If current flows through two lamps arranged: (a) in series, (b) in parallel, and the…
  6. The figure below shows a variable resistor in a dimmer switch. How would you turn the…
Long Answer Type Questions-pg-6
  1. What do you understand by the term potential difference?
  2. What is meant by saying that the potential difference between two points is 1 volt?…
  3. What is the potential difference between the terminals of a battery if 250 joules of work…
  4. What is the voltmeter? How is a voltmeter connected in the circuit to measure the…
  5. State whether a voltmeter has a high resistance or a low resistance. Give reason for your…
Very Short Answer Type Questions-pg-58
  1. State two factors on which the electrical energy consumed by an electrical appliance…
  2. Which one has a higher electrical resistance: a 100-watt bulb or a 60 watt bulb?…
  3. Name the commercial unit of electric energy.
  4. An electric bulb is rated at 220 V, 100 W. What is its resistance?…
  5. What is the SI unit of (i) electric energy, and (ii) electric power?…
  6. Name the quantity whose unit is (i) kilowatt, and (ii) kilowatt-hour.…
  7. Which quantity has the unit of watt?
  8. What is the meaning of the symbol kWh? Which quantity does it represent?…
  9. If the potential difference between the end of a wire of fixed resistance is doubled, by…
  10. An electric lamp is labeled 12 V, 36 W. This indicates that it should be used with a 12 V…
  11. What current will be taken by a 920 W appliance if the supply voltage is 230 V.…
Short Answer Type Questions-pg-58
  1. Define watt. Write down an equation linking watts, volts and amperes.…
  2. Define watt-hour. How many joules are equal to 1 watt-hour?
  3. How much energy is consumed when a current of 5 amperes flows through the filament (or…
  4. An electric bulb is connected to a 220 V power supply line. If the bulb draws a current of…
  5. In which of the following cases ore electrical energy is consumed per hour? (i) A current…
  6. An electric kettle rated at 220 V, 2.2 kW, works for 3 hours. Find the energy consumed and…
  7. In a house two 60 W electric bulbs are lighted for 4 hours, and three 100 W bulbs for 5…
  8. A bulb is rated as 250 V; 0.4 A Find its: (i) power, and (ii) resistance.…
  9. For a heater rated at 4 kW and 220 V, calculate: (a) the current, (b) the resistance of…
  10. An electric motor take 5 amperes current from a 220 volts supply line. Calculate the power…
  11. Which users more energy: a 250 W TV set in 1 hour or a 1200 W toaster in 10 minutes?…
  12. Calculate the power used in the 2 resistor in each of the following circuits: (i) a 6V…
  13. Two lamps, one rated 40 W at 220 V and the other 60 W and 220 V, are connected in parallel…
  14. An electric kettle connected to the 230 V mains supply draws a current of 10 A. Calculate:…
  15. A 2 kW heater, a 200 W TV and three 100 W lamps are all switched on from 6 p.m. to 10 p.m.…
  16. What is the maximum power in kilowatts of the appliance that can be connected safely to a…
  17. An electric fan runs from the 230 V mains. The current flowing through it is 0.4 A. At…
Long Answer Type Questions-pg-59
  1. What is meant by electric power? Write the formula for electric power in terms of…
  2. The diagram below shows a circuit containing a lamp L, a voltmeter and an ammeter. The…
  3. Define kilowatt-hour. How many joules are the in one kilowatt-hour?…
  4. Calculate the cost of operating a heater of 500 W for 20 hours at the rate of 3.90 per…
Multiple Choice Questions (mcqs)-pg-59
  1. When an electric lamp is connected to 12 V battery, it draws a current of 0.5 A. The power…
  2. The unit for expressing electric power is:A. volt B. joule C. coulomb D. watt…
  3. Which of the following is likely to be the correct wattage for an electric iron used in…
  4. An electric heater is rated at 2kW. Electrical energy costs 4 per kWh. What is the cost of…
  5. The SI unit of energy is:A. joule B. coulomb C. watt D. ohm-metre…
  6. The commercial unit of energy is:A. watt B. watt-hour C. kilowatt-hour D. kilo-joule…
  7. How much energy does a 100 W electric bulb transfer in 1 minute?A. 100 J B. 600 J C. 3600…
  8. An electric kettle for use on a 230 V supply is rated at 3000 W. For safe working, the…
  9. How many joules of electrical energy are transferred per second by a 6V; 0.5 A lamp?A. 30…
  10. At a given time, a house is supplied with 100 A at 220 V. How many 75 W, 220 V light bulbs…
  11. If the potential difference between the ends of a fixed resistor is halved, the electric…
Questions Based On High Order Thinking Skills (hots)-pg-60
  1. State whether an electric heater will consumer more electrical energy or less electrical…
  2. The table below shows the current in three different electrical appliances when connected…
  3. A boy noted the readings on his homes electricity meter on Sunday at 8 AM and again on…
  4. An electric bulb is rated as 10 W, 220V. How many of these bulbs can be connected in…
  5. Two exactly similar electric lamps are arranged (i) in parallel, and (ii) in series. If…
Very Short Answer Type Questions-pg-66
  1. How does the heat H produced by a current passing through a fixed resistance wire depend…
  2. If the current passing through a conductor is doubled, what will be the change in heat…
  3. Name two effects produced by electric current.
  4. Which effect of current is utilized in an electric light bulb?
  5. Which effect of current is utilized in the working of an electric fuse?…
  6. Name two devices which work on the heating effect of electric current.…
  7. Name two gases which are filled in filament type electric light bulbs.…
  8. Explain why, filament type electric bulbs are not power efficient.…
  9. Why does the connecting cord of an electric heater not glow hot while the heating element…
Short Answer Type Questions-pg-66
  1. Write down the formula for the heat produced when a current I is passed through a resistor…
  2. An electric iron of resistance 20 ohms draws a current of 5 amperes. Calculate the heat…
  3. State three factors on which the heat produced by an electric current depends. How does it…
  4. State and explain Joules law of heating.
  5. A resistance of 40 ohms and one of 60 ohms are arranged in series across 220 volt supply.…
  6. Why is an electric light bulb not filled with air? Explain why argon or nitrogen is filled…
  7. Explain why, tungsten is used for making the filaments of electric bulbs.…
  8. Explain why, the current that makes the heater element very hot, only slightly warms the…
  9. When a current of 4.0 A passes through a certain resistor for 10 minutes, 2.88 x 10^4 J of…
  10. A heating coil has a resistance of 200 . At what rate will heat be produced in it when a…
  11. An electric heater of resistance 8 takes a current of 15 A from the mains supply line.…
  12. A resistance of 25 is connected to a 12 V battery. Calculate the heat energy in joules…
  13. 100 joules of heat is produced per second in a 4 ohm resistor. What is the potential…
  14. Derive the expression for the heat produced due to a current I flowing for a time interval…
  15. How much heat will an instrument of 12 W produce in one minute if it is connected to a…
  16. The current passing through a room heater has been halved. What will happen to the heat…
  17. What is meant by the heating effect of current? Give two applications of the heating…
  18. Name the material which is used for making the filaments of an electric bulb.…
Multiple Choice Questions (mcqs)-pg-66
  1. The heat produced by passing an electric current through a fixed resistor is proportional…
  2. The current passing through an electric kettle has been doubled. The heat produced will…
  3. An electric fuse works on the:A. chemical effect of current B. magnetic effect of current…
  4. The elements of electrical heating devices are usually made of:A. tungsten B. bronze C.…
  5. The heat produced in a wire of resistance x when a current y flows through it in time z is…
  6. Which of the following characteristic is not suitable for a fuse wire?A. thin and short B.…
  7. In a filament type light bulb, most of the electric power consumed appears as :A. visible…
  8. Which of the following is the most likely temperature of the filament of an electric light…
  9. If the current flowing through a fixed resistor is halved, the heat produced in it will…
Questions Based On High Order Thinking Skills (hots)-pg-67
  1. The electrical resistivities of four materials P, Q, R and S are given below : P - 6.84…
  2. How does the wire in the filament of a light bulb behave differently to the other wires in…
  3. What property of the filament wire accounts for this difference?
  4. Two exactly similar heating resistances are connected (i) in series, and (ii) in parallel,…
  5. An electric iron is connected to the mains power supply of 220 V. When the electric iron…
  6. Which electric heating devices in your home do you think have resistors which control the…
Multiple Choice Questions (mcqs)-pg-6
  1. The work done in moving a unit charge across two points in an electric circuit is a…
  2. The device used for measuring potential differences is known as:A. potentiometer B.…
  3. Which of the following units could be used to measure electric charge?A. ampere B. joule…
  4. The unit for measuring potential difference is:A. watt B. ohm C. volt D. kWh…
  5. One coulomb charge is equivalent to the charge contained in:A. 2.6 X 10^19 electrons B.…
Very Short Answer Type Questions-pg-11
  1. By what name is the physical quantity coulomb /second called?
  2. What is the flow of charge called?
  3. What actually travels through the wires when you switch on a light?…
  4. Which particles constitute the electric current in a metallic conductor?…
  5. In which direction does conventional current flow around a circuit?…
  6. In which direction do electrons flow?
  7. Which of the following equation shows the correct relationship between electrical units? 1…
  8. What is the unit of electric current?
  9. How many milli-amperes are there in 1 ampere?
  10. How many micro-amperes are there in 1 ampere?
  11. Which of the two is connected in series: ammeter or voltmeter?
  12. Compare how an ammeter and a voltmeter are connected in a circuit.…
  13. What do the following symbols mean in circuit diagrams? (i) (ii)
  14. If 20 C of charge pass a point in a circuit in 1 s, what current is flowing?…
  15. A current of 4 A flows around a circuit for 10 s. How much charge flows past a point in…
  16. What is the current in a circuit if the charge passing each point is 20 C in 40 s ?…
  17. Fill in the following blanks with suitable words : (a) A current is a flow of…
Short Answer Type Questions-pg-11
  1. Name a device which helps to maintain potential difference across a conductor (say, a…
  2. If a potential difference of 10 V causes a current of 2 A to flow for 1 minute, how much…
  3. What is an electric current? What makes an electric current flow in a wire?…
  4. Define the unit of electric current (or Define ampere).
  5. What is an ammeter? How is it connected in a circuit? Draw a diagram to illustrate your…
  6. Write down the formula which relates electric charge, time and electric current.…
  7. A radio set draws a current of 0.36 A for 15 minutes. Calculate the amount of electric…
  8. Why should the resistance of : (a) an ammeter be very small? (b) a voltmeter be very…
  9. Draw circuit symbols for (a) fixed resistance (b) variable resistance (c) a cell (d) a…
  10. What is a circuit diagram? Draw the labelled diagram of an electric circuit comprising of…
  11. If the charge on an electron is 1.6 x I0-19 coulombs, how many electrons should pass…
  12. The p.d. across a lamp is 12 V. How many joules of electrical energy are changed into heat…
  13. In 10 s, a charge of 25 C leaves a battery, and 200 J of energy are delivered to an…
Long Answer Type Questions-pg-12
  1. Define electric current. What is the SI unit of electric current.…
  2. One coulomb of charge flows through any cross-section of a conductor in 1 second. What is…
  3. Which instrument is used to measure electric current? How should it be connected in a…
  4. What is the conventional direction of the flow of electric current? How does it differ…
  5. A flash of lightning carries 10 C of charge which flows for 0.01 s. What is the current?…
Multiple Choice Questions (mcqs)-pg-12
  1. The other name of potential difference is :A. ampereage B. wattage C. voltage D. potential…
  2. Which statement/statements is/are correct? 1) An ammeter is connected in series in a…
  3. Which unit could be used to measure current?A. Watt B. Coulomb C. Volt D. Ampere…
  4. If the current through a flood lamp is 5 A, what charge passes in 10 seconds?A. 0.5 C B. 2…
  5. If the amount of electric charge passing through a conductor in 10 minutes is 300 C, the…
Questions Based On High Order Thinking Skills (hots)-pg-12
  1. A student made an electric circuit shown here to measure the current through two lamps. A.…
  2. Draw a circuit diagram to show how 3 bulbs can be lit from a battery so that 2 bulbs are…
  3. An electric heater is connected to the 230 V mains supply. A current of 8A flows through…
  4. How many electrons are flowing per second past a point in a circuit in which there is a…

Very Short Answer Type Questions-pg-5
Question 1.

By what other name is the unit Joule⁄Coulomb called?


Answer:

Potential Difference is said to be 1 Volt between two points if 1 Joule of work is done in moving 1 coulomb of electric charge from one point to another.


Question 2.

Which of the following statements correctly defines a volt?
(a) A volt is a joule per ampere.
(b) A volt is a joule per coulomb.


Answer:

The second statement defines "volt" correctly.

Explanation: One Volt is the electric potential energy per unit charge, and is measured in joules per coulomb.


Question 3.

What do the letters p.d. stand for?


Answer:

p.d. stands for potential difference. Potential difference between two points in a circuit is the work done in moving unit charge (i.e. one coulomb) from one point to the other.



Question 4.

Which device is used to measure p.d.?


Answer:

Voltmeter is a device which is used to measure potential difference between two point in an electric ciruit.



Question 5.

What is meant by saying that the electric potential at a point is 1 volt?


Answer:

Electric Potential at a point being 1 volt means that 1 joule of work is done in moving 1 unit positive charge from infinity to that point


Question 6.

How much work is done when one coulomb charge moves against a potential difference of 1 volt?


Answer:

1 Joule of work is done when 1 coulomb of charge moves against a potential difference of 1 volt.

As potential difference =



Question 7.

What is the SI unit of potential difference?


Answer:

The SI unit of potential difference is volt, which is also known as Joule/Coulomb



Question 8.

How much work is done in moving a charge of 2 C across two points having a potential difference of 12 V?


Answer:

Given, charge = 2 C

Potential difference between two points = 12V


Work done (W) =?


We know that;


V = W/Q


Or, W = V × Q


Or, W = 12 V × 2 C = 24 J



Question 9.

What is the unit of electric charge?


Answer:

The SI unit of electric charge is coulomb (C).


Question 10.

Define one coulomb charge.


Answer:

One coulomb is that amount of electric charge that repels an equal and similar charge with a force of 9 × 109 N when placed in vacuum at a distance of one metre from it.



Question 11.

Fill in the following blanks with suitable words :

(a) Potential difference is measured in …………… by using a ……….. placed in ……… across a component.

(b) Copper is a good ………. plastic is an ………..


Answer:

(a) Potential difference is measured in volts by using a voltmeter placed in parallel across a component.

(b) Copper is a good conductor plastic is an insulator




Very Short Answer Type Questions-pg-18
Question 1.

Name the law which relates the current in a conductor to the potential difference across its ends.


Answer:

The law stating that the direct current flowing in a conductor is directly proportional to the potential difference between its ends. It is usually formulated as V = IR. This law is called Ohm's law.



Question 2.

Name the unit of electrical resistance and give its symbol.


Answer:

The ohm (symbol: Ω) is the SI derived unit of electrical resistance, named after German physicist Georg Simon Ohm.



Question 3.

Name the physical quantity whose unit is "ohm".


Answer:

The ohm (symbol: Ω) is the SI derived unit of electrical resistance.



Question 4.

What is the general name of the substances having infinitely high electrical resistance?


Answer:

Substances having infinite resistivity are called insulators Ex: wood.



Question 5.

Keeping the resistance constant, the potential difference applied across the ends of a component is halved.

By how much does the current change?


Answer:

As we know that

V = IR


So


As Current is directly proportional to potential difference


So Current is also become half



Question 6.

State the factors on which the strength of electric current flowing in a given conductor depends.


Answer:

The strength of electric current flowing in a given conductor depends upon the potential difference and resistance of the conductor.



Question 7.

Which has less electrical resistance: a thin wire or a thick wire (of the same length and same material) ?


Answer:

The larger the cross sectional area, the lower the resistance since the electrons have a larger area to flow through.



Question 8.

Keeping the potential difference constant, the resistance of a circuit is halved. By how much does the current change?


Answer:

As we know that

V = IR


As current is directly proportional to potential difference and inversely proportional to the resistance of the conductor. If resistance of a circuit is halved than the current will doubled.



Question 9.

A potential difference of 20 volts is applied across the ends of a resistance of 5 ohms. What current will flow in the resistance?


Answer:

As we know that

V = IR




Question 10.

A resistance of 20 ohms has a current of 2 amperes flowing in it. What potential difference is there between its ends?


Answer:

As we know that

V = IR


V = 2A × 20 ohms = 40 V



Question 11.

A current of 5 amperes flows through a wire whose ends are at a potential difference of 3 volts. Calculate the resistance of the wire.


Answer:

As we know that

V = IR




Question 12.

Fill in the following blank with a suitable word :

Ohm's law states a relation between potential difference and ........................


Answer:

Ohm's law states a relation between potential difference and current.




Short Answer Type Questions-pg-18
Question 1.

Distinguish between good conductors, resistors and insulators. Name two good conductors, two resistors and two insulators.


Answer:

On the basis of their electrical resistance, all the substance can be divided into three groups they are Good conductors, Resistors and Insulators.

Those substances which have very low electrical resistance are called as good conductors. A good conductor allows the electricity to flow through it easily. Silver is the best conductor of electricity. Copper and aluminum are also good conductors.


Those substances which have comparatively high resistance than conductors are known as resistors. The alloys like nichrome, manganin and constantan, all have quite high resistance, so these are resistors.


Those substances which have infinitely high electrical resistance are called insulators. E.g- rubber and wood.



Question 2.

Classify the following into good conductors, resistors and insulators:

Rubber, Mercury, Nichrome, Polythene, Aluminium, Wood, Manganin, Bakelite, Iron, Paper, Thermocol, Metal coin


Answer:

Those substances which have very low electrical resistance are called as good conductors like mercury, aluminum, iron, metal coin.

Those substances which have comparatively high resistance than conductors are known as resistors. The alloys like nichrome, manganin.


Those substances which have infinitely high electrical resistance are called insulators like rubber, polythene, wood, bakelite, paper, thermocol



Question 3.

What is Ohm's law? Explain how it is used to define the unit of resistance.


Answer:

Ohm's law states that the current through a conductor between two points is directly proportional to the voltage across the two points. Introducing the constant of proportionality, the resistance, one arrives at the usual mathematical equation that describes this relationship

I = V R


Where I is the current through the conductor in units of amperes, V is the voltage measured across the conductor in units of volts, and R is the resistance of the conductor in units of ohms. More specifically, Ohm's law states that the R in this relation is constant, independent of the current.


One ohm is defined as the resistance of an object when a current of one ampere flows through a an object with a potential difference of one volt.



Question 4.

What is meant by the "resistance of a conductor"? Write the relation between resistance, potential difference and current.


Answer:

The resistance of a conductor is the ratio of potential difference between the ends of a conductor to the current flowing through it, or the property of a conductor due to which it opposes the flow of current through it is called resistance of the conductor.

Potential differences(V) = Current (I) × Resistance(R)



Question 5.

When a 12 V battery is connected across an unknown resistor, there is a current of 2.5 mA in the circuit. Calculate the value of the resistance of the resistor.


Answer:

According to Question,

V = 12volt

I = 2.5mA= 2.5x10-3A

According to Ohms Law

V=IR




Question 6.

Define the of resistance (or Define the unit "ohm").


Answer:

The ohm is defined as a resistance between two points of a conductor when a constant potential difference of 1.0 volt, applied to these points, produces in the conductor a current of 1.0 ampere, the conductor not being the seat of any electromotive force.



Question 7.

What happens to the resistance as the conductor is made thinner?


Answer:

The resistance of the wire will increase if we made the conductor thinner because p (rho). l . Area is directly proportional to the radius of the wire.



Question 8.

Keeping the potential difference constant, the resistance of a circuit is doubled. By how much does the current changes?


Answer:

As we know that

V = IR


So, I = V/R


According to the questions


V= I'2R


So, I'= V/2R


I' = I/2


So current become half



Question 9.

Why do electricians wear rubber hand gloves while working with electricity ?


Answer:

Rubber is an insulator and does not allow the passage of electric current through it, so when electricians wear rubber gloves electric current cannot pass through them, so they don't get shock.



Question 10.

What p.d. is needed to send a current of 6 A through an electrical appliance having a resistance of 40 Ω


Answer:

As we know that

V = IR


Here I = 6A, R = 40Ω


So V = 6 × 40 = 240V



Question 11.

An electric circuit consisting of a 0.5 m long nichrome wire XY, an ammeter, a voltmeter, four cells of 1.5 V each and a plug key was set up.

(i) Draw a diagram of this electric circuit to study the relation between the potential difference maintained between the points 'X' and 'Y' and the electric current flowing through XY.

(ii) Following graph was plotted between V and I values:



What would be the value of ratios when the potential differences us 0.8 V, 1.2 V and 1.6 V respectively?

What conclusion does you drawn from the values?

(iii) What is the resistance of the wire?


Answer:

(i)

(ii) From graph we find that current values corresponding to potential difference of 0.8V, 1.2V and 1.6V respectively are 0.3 A, 0.45A and 0.6A.






From these values we conclude that ration V/I is a constant i.e., V α I. It means that Ohm’s law is being followed.


(iii) The resistance of the wire is equal to the ratio of potential difference applied and the current passing through it.





Long Answer Type Questions-pg-19
Question 1.

What is the ratio of potential difference and current known as?


Answer:

Resistance is the ratio of potential difference across a component to the current flowing through it, it is measure in ohms.



Question 2.

The values of potential difference V applied across a resistor and the corresponding values of current I flowing in the resistor are given below :


Plot a graph between V and I, and calculate the resistance of the resistor.


Answer:

The plot between voltage and current is called IV characteristics. The voltage is plotted on y- axis. The values of the current for different values of the voltage are shown in the given table

The plot between voltage and current is called IV characteristic. The voltage is plotted on x-axis and current is plotted on y-axis. The values of the current for different values of the voltage are shown in the given table




Question 3.

Name the law which illustrated by the above V-I graph.


Answer:

From these values we conclude that ration V/I is a constant i.e., V α I. It means that Ohm’s law is being followed.



Question 4.

Write down the formula which states the relation between potential difference, current and resistance.


Answer:

The formula is given by Ohm which states the relation between potential difference, current and resistance.

Potential difference = Current × Resistance



Question 5.

The potential difference between the terminals of an electric iron is 240 V and the current is 5.0 A. What is the resistance of the electric iron?


Answer:

Here, V=240 volt, I= 5 A, R=?
According to Ohms Law:-
V= IR



Multiple Choice Questions (mcqs)-pg-19
Question 1.

The p.d. across a 3Ω resistor is 6 V. The current flowing in the resistor will be:
A.

B. 1 A

C. 2 A

D. 6 A


Answer:

As we know that


Question 2.

A car headlight bulb working on a 12 V car battery draws a current of 0.5 A. The resistance of the light bulb is:
A. 0.5 Ω

B. 6 Ω

C. 12 Ω

D. 24 Ω


Answer:

As we know that

Ω


Question 3.

An electrical appliance has a resistance of 25 Ω. When this electrical appliance is connected to a 230 V supply line, the current passing through it will be :
A. 0.92 A

B. 2.9 A

C. 9.2 A

D. 92 A


Answer:


Question 4.

When a 4Ω resistor is connected across the terminals of a 12 V battery, the number of coulombs passing through the resistor per second is :
A. 0.3

B. 3

C. 4

D. 12


Answer:

R= 4 ohms

V= 12 V


I= 12/4 =3A


I=Q/t


Q=3 C


Question 5.

Ohm’s law gives a relationship between:
A. current and resistance

B. resistance and potential difference

C. potential difference and electric charge

D. current and potential difference


Answer:

Ohm’s law gives a relationship between current and potential difference.


Question 6.

The unit of electrical resistance is :
A. ampere

B. volt

C. coulomb

D. ohm


Answer:

Ohm is the S.I unit of electrical resistance.


Question 7.

The substance having infinitely high electrical is called:
A. conductor

B. resistor

C. superconductor

D. insulator


Answer:

The substance having infinitely high electrical is called conductor.


Question 8.

Keeping the potential difference constant, the resistance of a circuit is halved. The current will become:
A. one-fourth

B. four times

C. half

D. double


Answer:

Current is inversely proportional to resistance.


Question 9.

Keeping the potential difference constant, the resistance of a circuit is halved. The current will become:
A. one-fourth

B. four times

C. half

D. double


Answer:

Double



Questions Based On High Order Thinking Skills (hots)-pg-20
Question 1.

An electric room heater draws a current of 2.4 A from the 120 V supply line. What current will this room heater draw when connected to 240 V supply line?


Answer:

I=2.4A

potential difference = 120V:


R= V/I


R= 120/2.4 =50 ohms


Since resistance remains constant therefore if potential difference = 240V


then, I= V/R = 240/50 = 4.8A



Question 2.

Name the electrical property of a material whose symbol is “omega”.


Answer:

The electrical property of a material whose symbol is “omega is resistance.



Question 3.

The graph between V and I for a conductor is a straight line passing through the origin.
(a) Which law is illustrated by such a graph?
(b) What should remain constant in a statement of this law?


Answer:

(a) The I-V graph of the metallic conductor is a straight line pass through the origin. They obey Ohm’s Law, having a resistance that is independent of current. (Ohmic conductors). The variation of current vs voltage is shown below:

(b) Temperature remains constant in ohms law.


Question 4.

A p.d. of 10 V is needed to make a current of 0.02 A flow through a wire. What p.d. is needed to make a current of 250 mA flow through the same wire?


Answer:

Here I1 = 0.02 amp, V1 = 10 volt

V=IR



Again


Here I2 = 250mA=0.25A, R = 500


V2=I2R =0.25×500=125V



Question 5.

A current of 200 mA flows through a 4 KΩ resistor. What is the p.d. across the resistor?


Answer:

Given,

I = 200mA = 200 × 10-3A


R = 4000 ohms


So, V = 200 × 10-3A × 4×103


V= 800 volts




Very Short Answer Type Questions-pg-26
Question 1.

What happens to the resistance as the conductor is made thicker?


Answer:

The resistance of the wire will decrease if we made the conductor thicker because R (resistance) = p (rho). l (length) / A (area of cross-section or thickness of wire). Area is directly proportional to the radius of the wire.



Question 2.

If the length of a wire is doubled by taking more of wire, what happens to its resistance?


Answer:

As the resistance of a wire is directly proportional to length of the conducting wire. If its length is doubled resistance also doubled.



Question 3.

On what factors does the resistance of a conductor depend?


Answer:

There are four factors that influence the resistance in a conductor. Thickness (cross sectional area of the wire), length, and temperature. The fourth factor is the conductivity of the material that is used.



Question 4.

Name the material which is the best conductor of electricity.


Answer:

Silver is the best conductor of electricity.



Question 5.

Which among iron and mercury is a better conductor of electricity?


Answer:

Mercury is a better conductor than iron as it has lower resistivity than iron.



Question 6.

Why are copper and aluminium wires usually used for electricity transmission?


Answer:

Copper and aluminium wires usually used for electricity transmission because copper and aluminium are very good conductors of electricity having very low electric resistivity (or resistance).



Question 7.

Name the material which is used for making the heating element of an electric iron.


Answer:

Nichrome wire is used as a heating element because it is very stable, even at high temperatures.



Question 8.

What is nichrome? State its one use.


Answer:

Nichrome (NiCr, nickel-chrome, chrome-nickel, etc.) generally refers to any alloy of nickel, chromium, and often iron and/or other elements or substances. Nichrome alloys are typically used in resistance wire. They are also used in some dental restorations (fillings) and in other applications.



Question 9.

Give two reasons why nichrome alloy is used for making the heating elements of electrical appliances.


Answer:

Firstly nicrome is an alloy, it has a higher resistivity and consequently a higher resistance.Therefore, it will resist the flow of charges more and lead to development of heat faster.

Secondly alloys like nichrome don't oxidise,i.e burn easily at high temperatures.



Question 10.

Why are the coils of electric irons and electric toasters made of an alloy rather than a pure metal?


Answer:

The resistivity of an alloy is generally higher than that of its constituent metals. Alloys do not oxidise readily at high temperatures.



Question 11.

Which has more resistance:

(a) a long piece of nichrome wire or a short one?

(b) a thick piece of nichrome wire or a thin piece?


Answer:

(a) The resistance of a wire depends on its dimensions as well as on the conducting ability of the material from which it is made. A long wire has more resistance than a short one.

(b) A thin wire has more resistance than a thick one.



Question 12.

How does the resistance of a pure metal change if its temperature decreases?


Answer:

If temperature of a metal decrease the resistance will also decrease.



Question 13.

How does the presence of impurities in a metal affect its resistance?


Answer:

The presence of impurities in a metal increase the resistance of the metal.



Question 14.

Fill in the following blanks with suitable words :

Resistance is measured in ............. The resistance of a wire increases as the length ...............; as the temperature ................; and as the cross-sectional area ............


Answer:

Resistance is measured in Ohms. The resistance of a wire increases as the length increases, as the temperature increases, and as the cross-sectional area decreases.




Short Answer Type Questions-pg-26
Question 1.

What do you understand by the "resistivity" of a substance?


Answer:

The electrical resistivity of a material is also known as its specific electrical resistance. It is a measure of how strongly a material opposes the flow of electric current. A definition of resistivity is the electrical resistance per unit length and per unit of cross-sectional area.



Question 2.

A wire is 1.0 m long, 0.2 mm in diameter and has a resistance of 10Ω. Calculate the resistivity of its material?


Answer:

Given:
The length of the wire, l=1m.
Diameter of wire, D = 0.2mm
∴ Radius of Wire,r=0.1mm= 0.1x10-3m
Mathematically, the resistivity is defined as


Question 3.

Write down an expression for the resistance of a metallic wire in terms of the resistivity.


Answer:

Resistance of a metallic wire with resistivity of length l and cross sectional area A is



Question 4.

What will be the resistance of a metal wire of length 2 metres and area of cross-section 1.55 x 10-6 m2, if the resistivity of the metal be 2.8 x 10-8 Ω m ?


Answer:




Question 5.

Give two examples of substances which are good conductors of electricity. Why do you think they are good conductors of electricity?


Answer:

Metals are typically good conductors because of their ability to donate electrons. Most metal atoms have one or two valence electrons, which means that they want to give these away in order to become more stable. Silver and copper are good conductors of electricity because they have free electrons available for conduction.



Question 6.

Calculate the resistance of a copper wire 1.0 km long and 0.50 mm diameter if the resistivity of copper is 1.7×10-8 Ω m.


Answer:

According to Question,

Length of Wire= 1km=1000m

Diameter of Wire=0.50mm

∴ Radius of wire= 0.25mm=0.25x10-3m

Resistivity(ρ)=1.7x10-8Ωm



Question 7.

Will current flow more easily through a thick wire or a thin wire of the same material when connected to the same source? Give reason for your answer.


Answer:

The current will flow more easily through thick wire. It is because the resistance of a conductor is inversely proportional to its area of cross - section. If thicker the wire, less is resistance and hence more easily the current flows.



Question 8.

How does the resistance of a conductor depend on :

(a) length of the conductor?

(b) area of cross-section of the conductor?

(c) temperature of the conductor?


Answer:

a) Resistance of a conductor is directly propotional to the length of conductor.

b) Resistance of a conductor is inversely proportional to the area of cross section of the conductor.


c) Resistance of a conductor depends upon the nature of the material of the conductor. Resistance of a conductor increases on raising the temperature and decreases on lowering the temperature.



Question 9.

Give one example to show how the resistance depends on the nature of material of the conductor.


Answer:

If we connect different type of wire (nichrome, copper etc) in ac ircuit made using a battery, ammeter. We will see that the ammeter shows different reading in each case i.e. resistance differes as nature of material changes.



Question 10.

Calculate the resistance of an aluminium cable of length 10 km and diameter 2.0 mm if the resistivity of aluminium is 2.7 x 10-8Ωm.


Answer:



Question 11.

What would be the effect on the resistance of a metal wire of :

(a) increasing its length?

(b) increasing its diameter?

(c) increasing its temperature?


Answer:

(a) The resistance of a metal wire will increase as it is directly proportional to length.

(b) The resistance of a metal wire decrease as it is inversely proportional to the area of the wire.


(c) Resistance of a conductor depends upon the nature of the material of the conductor. Resistance of a conductor increases on raising the temperature and decreases on lowering the temperature



Question 12.

How does the resistance of a wire vary with its :

(a) area of cross-section?

(b) diameter?


Answer:

(a) The resistance of a metal wire is inversely proportional to the area of the wire, so if the area increase resistance decrease.

(b) If diameter increase than area increase and the resistance decreases.



Question 13.

How does the resistance of a wire change when:

(i) its length is tripled?

(ii) its diameter is tripled?

(iii) its material is changed to one whose resistivity is three times?


Answer:

(i) The resistance of a wire is directly proportional to its length and inversely proportional to its area (or square of inverse of diameter). When the length is tripled then its resistance will also becomes three times

(ii) If the diameter get tripled then its resistance becomes one ninth of the initial resistance.


(iii) If the material is changed to one whose resistivity is three times then again its resistance will be three times



Question 14.

Calculate the area of cross section of a wire if its length is 1.0 m, its resistance is 23Ω and the resistivity of the material of the wire is 1.84 x 10-6 Ωm.


Answer:

Here length of the wire = 1 m

Resistance of the wire = 23 ohms


Resistivity of the wire =1.84 × 10-6 ohm meter


As we know that






Long Answer Type Questions-pg-27
Question 1.

Define resistivity. Write an expression for the resistivity of a substance. Give the meaning of each symbol which occurs in it.


Answer:

By definitions the resistivity of a material of a conductor is the resistance offered by a unit length and unit cross-section of the conductor. It is intrinsic property of material.

The expression for the resistivity is



Where m = mass of the electron


N= number density of electron


E = charge on the electron


Ƭ = relaxation time.



Question 2.

State the SI unit of resistivity.


Answer:

Resistivity is commonly represented by the Greek letter ρ (rho). The SI unit of electrical resistivity is the ohm-metre (Ω⋅m).



Question 3.

Distinguish between resistance and resistivity.


Answer:

The following points state the differences between Resistance and Resistivity:


Question 4.

Name two factors on which the resistivity of a substance depends and two factors on which it does not depend.


Answer:

Resistivity of a substance depends on the nature of the substance and its temperature. It does not depend on the length or thickness of the conductor.



Question 5.

The resistance of a metal wire of length 1 m is 26 Ω at 20° C. If the diameter of the wire is 0.3 mm, what will be the resistivity of metal at that temperature?


Answer:

We know,

According to Question,
Length of the conductor(In this case wire) = 1m
Diameter of the wire= 0.3mm

Resistance of the conductor(In this case wire)= 26Ω



Multiple Choice Questions (mcqs)-pg-27
Question 1.

The resistance of a wire of length 300 m and cross-section area 1.0 mm2 made of material of resistivity 1.0 x 10-7 Ω m is :
A. 2 Ω

B. 3 Ω

C. 20 Ω

D. 30 Ω


Answer:


Question 2.

When the diameter of a wire is doubled, its resistance becomes:
A. double

B. four times

C. one-half

D. one-fourth


Answer:

When the diameter of a wire is doubled, its resistance becomes one-fourth.


Question 3.

If the resistance of a certain copper wire is 1Ω, then the resistance of a similar nichrome wire will be about:
A. 25 Ω

B. 30 Ω

C. 60 Ω

D. 45 Ω


Answer:

Alloy of nickel, chromium, manganese and iron having a resistivity of about 60 times more than that of copper.


Question 4.

If the diameter of a resistance wire is halved, then its resistance becomes:
A. four times

B. half

C. one-fourth

D. two times


Answer:

If the diameter of a resistance wire is halved, then its resistance becomes four times.


Question 5.

The resistivity of a certain material is 0.6 Ωm. The material is most likely to be:
A. an insulator

B. a superconductor

C. a conductor

D. a semiconductor


Answer:

The resistivity of semiconductor like silicon and germanium is in between those of conductor and insulator.


Question 6.

When the area of cross-section of a conductor is doubled, its resistance becomes:
A. double

B. half

C. four times

D. one-fourth


Answer:

When the area of cross-section of a conductor is doubled, its resistance becomes


Question 7.

The resistivity of copper metal depends on only one of the following factors. This factor is :
A. length

B. thickness

C. temperature

D. area of cross-section


Answer:

The resistivity of copper metal depends on temperature.


Question 8.

If the area of cross-section of a resistance wire is halved, then its resistance becomes :
A. one-half

B. 2 times

C. one-fourth

D. 4 times


Answer:

If the area of cross-section of a resistance wire is halved, then its resistance becomes twice.



Questions Based On High Order Thinking Skills (hots)-pg-27
Question 1.

A piece of wire of resistance 20 Ω is drawn out so that its length is increased to twice its original length. Calculate the resistance of the wire in the new situation.


Answer:

We known that


Now if we stretch the same wire in such a way that its length get double. At the same time its area of cross-section decrease to half. So that




Question 2.

The electrical resistivities of three materials P, Q and R are given below :

P 2.3 x 103 Ω m

Q 2.63 x 10-8 Ω m

R 1.0 x 1015 Ω m

Which material will you use making (a) electric wires (b) handle for soldering iron, and (c) solar cells? Give reasons for your choices.


Answer:

(a) Material Q with resistivity 2.63×10-8 ohm-m can be used for making electric wires because it has very low resistivity.

(b) Material R with resistivity 1.0 ×1015 ohm-m can be used for making handle of soldering iron because it has very high resistivity.


(c) Material P with resistivity 2.3 × 103 ohm-m can be used for making solar cell because it is a semiconductor.



Question 3.

The electrical resisitivities of four materials A, B, C and D are given below :

A) 110 x 10-8 Ω m

B) 1.10 x 1010 Ω m

C) 01.0 x 10-8 Ω m

D) 2.3 x 103 Ω m

Which material is : (a) good conductor (b) resistor (c) insulator, and (d) semi-conductor ?


Answer:

(a) Good conductor = C (10×10-8 ohm-m)

(b) Resistor= A (110×10-8 ohm-m)


(c) Insulator= B (1×1010 ohm-m)


(d) Semiconductor= D (2.3 ×103 ohm-m)



Question 4.

The electrical resistivities of five substances A, B, C, D and E are given below :

A) 5.20 x 10-8 Ω m

B) 2.60 x 10-8 Ω m

C) 10.0 x 10-8 Ω m

D) 1.70 x 10-8 Ω m

(a) Which substance is the best conductor of electricity? Why?

(b) Which one is better conductor: A or C? Why?

(c) Which substance would you advice to be used for making heating elements of electric irons? Why?

(d) Which two substances should be used for making electric wires? Why?


Answer:

(a) E is the best conductor of electricity due to its least electrical resistivity.

(b) C, because its resistivity is lesser than that of A.


(c) B, because it has highest electrical resistivity


(d) C and E, because of their low electrical resistivities.




Short Answer Type Questions-pg-5
Question 1.

What is meant by conductors and insulators? Give two examples of conductors and two of insulators.


Answer:

Those substance which allow electric current to pass through them easily is called conductors. Examples. Iron, copper, zinc e.t.c

Those substance which do not allow electric current to pass through them is called Insulators. Examples. Mica, Air, Paper, e.t.c



Question 2.

Which of the following are conductors and which are insulators?

Sulphur, Silver, Copper, Cotton, Aluminium, Air, Nichrome, Graphite, Paper, Porcelain, Mercury, Mica, Bakelite, Polythene, Manganin.


Answer:

Conductor allow electric current to pass through them, so here conductors are Silver, Copper, Aluminum, Nichrome, Graphite, Mercury, Manganin and insulator do not allow current to pass through them, so here insulators are Sulphur, Cotton, Air, Paper, Porcelain, Mica, Bakelite, Polythene



Question 3.

What do you understand by the term “electric potential”? (or potential) at a point? What is the unit of electric potential?


Answer:


Electronic Potentiol or Potential is defined as the work done in moving a unit positive charge from infinity to a particular point. It is denoted by V and its unit is "Volt".


Question 4.

State the relation between potential difference, work done and charge moved.


Answer:

Electric potential difference is known as voltage, which is equal to the work done per unit charge to move the charge between two points against static electric field.



Question 5.

Calculate the work done in moving a charge of 4 coulombs from a point at 220 volts to another point at 230 volts.


Answer:

Here charged moved = 4 C

Potential difference = V2 – V1 =230V – 220V = 10 V


So as we know that


Work done = Potential difference × Charge moved


= 10 × 4 = 40 joule


So, the work done in moving a charge of 4 coulombs from a point at 220 volts to another point at 230 volts is 40 Joule.



Question 6.

Name a device that helps to measure that potential difference across a conductor.


Answer:

Voltmeter is an apparatus used to measure the potential difference or electric potential difference between two points in an electric circuit.



Question 7.

How much energy is transferred by a 12 V power supply to each coulomb of charge which it moves around a circuit?


Answer:

Here potential difference = 12 V

Charge moved = 1 C


Work done = Potential difference × Charge moved


= 12 × 1 = 12 joule


So 12 J energy is transferred by a 12 V power supply to each coulomb of charge which it move around a circuit.




Very Short Answer Type Questions-pg-37
Question 1.

Give the law of combination of resistances in series.


Answer:

When resistors are joined from end to end, it is called in series. In this case, the total resistance of the system is equal to the sum of the resistance of all the resistors in the system.



Question 2.

If five resistances, each of value 0.2 ohm, are connected in series, what will be the resultant resistance?


Answer:

According to the law of combination of series connection

We know that




Question 3.

State the law of combination of resistances in parallel.


Answer:

When resistors are joined in parallel, the reciprocal of total resistance of the system is equal to the sum of reciprocal of the resistance of resistors.



Question 4.

If 3 resistances of 3 ohm each are connected in parallel, what will be their total resistance?


Answer:

We know that in parallel combination, the reciprocal of total resistance is



Question 5.

How should the two resistances of 2 ohms each be connected so as to produce an equivalent resistance of 1 ohm?


Answer:

According to the question the resultant resistance is less than the individual resistances, so the resistances should be connected in parallel not in series.



Question 6.

Two resistances X and Y are connected turn by turn : (i) in parallel, and (ii) in series. In which case the resultant resistance will be less than either of the individual resistance?


Answer:

The resultant resistance will be less than either of the individual resistance in parallel connection.



Question 7.

What possible values of resultant resistance one can get by combining two resistances, one of value 2 ohm and the other 6 ohm?


Answer:

Here, R1=2 ohm, R2=6 ohm

When the resistors are arranged in parallel combination we get,



Again, when the resistors are arranged in series combination we get


R = R1 + R2 = 2 + 6 = 8ohm



Question 8.

Show how you would connect two 4 ohm resistors to produce a combined resistance of (a) 2 ohms (b) 8 ohms.


Answer:

When two 4 ohm resistors are connected in parallel then the equivalent resistance will be

1/Requivalent = 1/4 + 1/4 = 2/4 = 1/2


Or R = 2 ohms


When two 4 ohm resistors are connected in series then the equivalent resistance will be


Requivalent = 4 ohm + 4 ohm = 8 ohm



Question 9.

Which of the following resistor arrangement, A or B, has the lower combined resistance?





Answer:

Equivalent resistance in arrangement A is 10 ohm as this is in series.

And in arrangement B both 10 ohm and 1000 ohm are in parallel.


So, combined resistance of arrangement B is caculated as follows:


1/R = 1/10 + 1/1000 = (100+1)/1000


R = 1000/101 = 9.9 ohm


So, arrangement in B has lower combined resistance.



Question 10.

A wire that has resistance R is cut into two equal piece. The two parts are joined in parallel. What is the resistance of the combination?


Answer:

As the wire is cut into two equal pieces. So resistance of each part is R/2.

The two parts are joined in parallel. So the equivalent resistance will be


1/Requivalent = 2/R + 2/R


1/Requivalent= 4/R


Requivalent = R/4



Question 11.

Calculate the combined resistance in each case:







Answer:

(a) Here both resistance i.e. R1 = 500 ohm, R2 = 1000 ohm are connected in series

So equivalent resistance R = R1 + R2 = 500 + 1000 = 1500ohm.


(b) Here both resistance i.e. R1=2ohm, R2=2ohm are connected in parallel


So equivalent resistance


1/R=1/R1+1/R2


1/R=1/2+1/2


R=1ohm


(c) Here resistance R1=4ohm and R2=4ohm are in parallel and R3=3ohm are in series


According to the given figure,


1/R=1/R1+1/R2


1/R=1/4+1/4


R=2ohm


Total resistance of the given circuit =R+R3


=2+3=5ohm



Question 12.

Find the current in each resistor in the circuit shown below:



Answer:

Let R1=6Ὠ, R2=4Ὠ and V=24V

As the two resistances are connected in parallel. So the current across R1 will be



And the Current across R2 will be





Short Answer Type Questions-pg-38
Question 1.

Explain with diagrams what is meant by the “series combination” and “parallel combination” of resistances. In which case the resultant resistance is: (i) less, and (ii) more, than either of the individual resistances?


Answer:

In a circuit, resistors are connected end-to-end are said to be in series, if the same current

exists in all of them through a single path. When resistances are connected in series, their equivalent resistance is equal to the sum of the individual resistances.


R = R1 + R2 + R3 + - - - - -



When resistances are connected in parallel, the reciprocal of their equivalent resistance is equal to the sum of the reciprocals of the individual resistances.


1/R = 1/R1 + 1/R2 + 1/ R3 + - - - - -



The resultant resistance is less than either of the individual resistances.



Question 2.

A battery of 9 V is connected in series with resistors of 0.2 Ω, 0.3 Ω, 0.4 Ω, 0.5 Ω and 12 Ω. How much current would flow through the 12 Ω resistor?


Answer:

Here a battery of 9 V is connected in series with resistors of R1=0.2ohm, R2=0.4ohm, R3=0.3ohm, R4=0.5ohm, R5=12ohm,

So the resultant resistance = R1 + R2 + R3 + R4 + R5


R = 0.2+0.4+0.3+0.5+12=13.4ohm


As we know that


V= IR


Thus the current flow through 12ohm resistance will be equal to the current flowing across the whole circuit as in case of Series Connection same current flows through each resistance


∴ I = V/R


I = 9/13.4


I = 0.67amp.


Question 3.

An electric bulb of resistance 20Ω and a resistance wire of 4 Ω are connected in series with a 6 V battery. Draw the circuit diagram and calculate:

(a) total resistance of the circuit

(b) current through the circuit.

(c) potential difference across the electric bulb.

(d) potential difference across the resistance wire.


Answer:

(a) Here resistance are connected in series


Total resistance of the circuit = R1+R2=20+4=24Ω(ohm)


(b) According to ohm’s law.


V=IR


Therefore,


6V=I x 24Ω(ohm)


I=6 V /24 ohm = 0.25A(Ampere)


(c) Potential difference across the electric bulb


V1= IR1=0.25 X 20=5V


(d) Potential difference across the resistance wire


V2= IR2=0.25X4=1V


Question 4.

Three resistors are connected as shown in the diagram.



Through the resistor 5 ohm, a current of 1 ampere is flowing

(i) What is the current through the other two resistors?

(ii) What is the p.d. across AB and across AC?

(iii) What is the total resistance?


Answer:

(i) According to the figure.


R1 and R2 are connected in parallel.


Let I be the current following in the circuit, which is equal to 1A. After passing through the resistance R3


Current I is divided into two part say I1 and I2



(ii) Potential difference across AB = IR3 = 1 x 5 = 5V


As R1 and R2 are connected in parallel


So the equivalent resistance 1/Requivalent = 1/R1 +1/R2


1/Requivalent= 1/10 + 1/15


Requivalent= 6 ohm


Total resistance in the circuit


Rtotal= = 5+6 = 11 ohm


Potential difference across AC = IR = 1x11 = 11V



Question 5.

For the circuit shown in the diagram below:

What is the value of:



(i) current through 6 Ω resistor?

(ii) potential difference across 12 Ω resistor?


Answer:

The resistance of 6 ohm and 3 ohm are connected in series. Therefore, their net resistance can be calculated as

R= R1+R2


Here, R1= 6ohm


R2= 3ohm


so R= 6ohm +3ohm= 9 ohm


The current through 6 ohm resistor= current through line 1 = I= V/R =4/9=0.44V


(ii) The resistance of 12ohm and 3 ohm are connected in series. Therefore, their net resistance can be calculated as


R= R1+R2


Here, R1= 12ohm


R2= 3ohm


so R= 12ohm +3ohm= 15 ohm


The current through them = I= V/R =4/15


Potential difference across 12 ohm resistor= 4/15 ×12 =3.2



Question 6.

Two resistors, with resistances 5 Ω and 10 Ω respectively are to be connected to a battery of emf 6 V so as to obtain:

(i) minimum current flowing

(ii) maximum current flowing

(a) How will you connect the resistances in each case?

(b) Calculate the strength of the total current in the circuit in the two cases.


Answer:

(i) For obtaining minimum current, the two resistors should be connected in parallel.

(ii) For obtaining maximum current, the two resistors should be connected in series.


(a) R1= 5 ohm, R2= 10 ohm and V = 6 V


For parallel combination




R= 10/3 ohm


Total current in the circuit



For series combination


R=R1+R2


R=5+10=15 ohm


Total current in the circuit.



(b) So the current in each case is 1.8 A and 0.4A in parallel and series circuit respectively.



Question 7.

The circuit diagram given below shows the combination of three resistors R1, R2 and R3 :



Find:

(i) total resistance of the circuit.

(ii) total current flowing in the circuit.

(iii) the potential difference across R1.


Answer:

(i) As shown in the figure, the resistor R2 and R3 are connected in parallel. Their total resistance is given by


Here R2= 3 ohm and R3=6 ohm


So


R =2 ohm


The resistance R1 is in series with the equivalent resistance which are parallel.


Total resistance of the circuit = 2+4ohms = 6ohms


(ii) Total current flowing through the circuit=Potential difference/total resistance


Here p.d =12V, and total resistance = 6 ohms


I = 12/6 = 2amps


(iii) The potential difference across R1.


V = IR1=2×4 =8V



Question 8.

In the circuit diagram given below, the current flowing across 5 ohm resistor is 1 amp. Find the current flowing through the other two resistors.



Answer:

Given I= 1 A (Across 5 ohm)

R= 5 ohm


The potential drop across AB, V = I × R


or V= 5 ohm × 1A = 5V


In a parallel circuit, the potential difference across the ends of all resistors remains the same. Therefore the potential difference across 4 ohm and 10 ohm will be 5 V


The current flowing through the 4 ohm resistance


= V/R = 5/4 = 1.25 A


Current through 10 ohm resistor = V/R = 5/10 = 0.5 A



Question 9.

A resistor has a resistance of 176 ohms. How many of these resistors should be connected in parallel so that their combination draws a current of 5 amperes from a 220 volt supply line?


Answer:

We Know according to Ohms Law that:-

V= IR
Where V= Voltage/Potential Difference
I= Current
R=Resistance

According to Question :-V= 220V
I= 5A
∴ Total Resistance of the Combination is :-

Suppose x resistors should be connected in parallel to draw a current of 5A

for parallel combination:-

1/R = 1/R1 +1/R2+ 1/R3........1/RX(Where Resistance of each Resistor is 176Ω)

According to the question,

RCombination=176/x

⇒44=176/x( We know from above the total resistance of combination)

x=176/44=4 resistor

Hence 4 resistors each of 176Ω should be connected in parallel so as to draw the current of 5A from a 220 volt supply line.


Question 10.

An electric heater which is connected to 220 V supply line has two resistance coils A and B of 24 Ω resistance each. These coils can be used separately (one at a time), in series or in parallel. Calculate the current drawn when:

(a) only one coil A is used.

(b) coils A and B are used in series

(c) coils A and B are used in parallel.


Answer:

(a) When only one coil, A is used

V= IR


220= 24I


I=9.2


(b) When coils A and B are used in series.


Total resistance R= RA +RB = 24+24 = 48ohms


I = V/R = 220/48


=4.58amps


(c) When coils A and B are used in parallel.




R=12ohms


I = V/R = 220/12


=18.33amps



Question 11.

In the circuit diagram given below five resistances of 10 Ω, 40 Ω, 30 Ω, 20 Ω and 60 Ω are connected as shown to a 12 V battery.



Calculate:

(a) total resistance in the circuit.

(b) total current flowing in the circuit.


Answer:

(a) when three resistors are connected in parallel, the net resistance can be obtained as followed


The resistance of 30 ohm, 20 ohm and 60 ohm are connected in parallel. therefore, the net resistance R will be


----------------------- (1)


The resistance of 10 ohm and 60 ohm are connected in parallel. Therefore, the net resistance R will be




Now R1 and R2 are connected in series


So total resistance in the circuit is R = 10+8 =18 ohm


(b) Total current flowing in the circuit




Question 12.

In the circuit diagram given below, three resistors, R1, R2 and R3 of 5 Ω, 10 Ω and 30 Ω respectively are connected as shown.



Calculate :

(a) current through each resistor.

(b) total current in the circuit.

(c) total resistance in the circuit.


Answer:

(a) Let I1, I2 and I3 be the current flowing through the resistors of 5 ohm, 100 ohm and 30 ohm, respectively.

According to ohm's law V=IR


Here V= 12 V and R = 5 ohm


I = V/R


Current through R1 = V/R1 = 12/5 = 2.4 A


Current through R2 = V/R2 = 12/10 = 1.2 A


Current through R3 = V/R2 = 12/30 = 0.4 A


(b) Total current in the circuit =Current through R1 +Current through R2 +Current through R3


= 2.4 + 1.2 + 0.4 = 4 A


(c) Total resistance in the circuit = R




R= 3 ohm



Question 13.

A p.d. of 4 V is applied to two resistors of 6 Ω and 2 Ω and 6 Ω connected in parallel. Calculate:

(a) the combined resistance

(b) the current flowing in the main circuit

(c) the current flowing in the 6Ω resistor.


Answer:

(a) When two resistors are connected in series, their resultant resistance is given by

R= R1+R2


Here R1 = 6 ohm


R2= 2 ohm


Combined resistance, R = R1 + R2 = 6+2 = 8 ohm


(b) We know that the current, I= V/R


Here R = 8 ohm


V= 4V


I = 4/8 = 0.5 amp


(c) Potential difference across 6ohm resistor V= I x R1 = 0.5 x 6 = 3 V



Question 14.

A p.d. of 6 V is applied to two resistors of 3Ω and 6Ω connected in parallel. Calculate:

(a) the combined resistance

(b) the current flowing in the main circuit

(c) the current following in the 3Ω resistor


Answer:

(a) ∵ the 3 Ω and 6 Ω resistance are in parallel, hence the eq. resistance =

(b) I = V/Req = 4/2 = 2A


(c) Current Flowing in 3 Ω resistor =



Question 15.

In the circuit shown below, the voltmeter reads 10 V.



(a) What is the combined resistance?

(b) What current flows?

(c) What is the p.d. across 2 Ω resistor?

(d) What is the p.d. across 3 Ω resistor?


Answer:

When two resistors are connected in series, their resultant resistance is given by

R= R1+R2


Here R1 = 2 ohm


R2= 3 ohm


Combined resistance, R = R1 + R2 = 2+3 = 5 ohm


(b) We know that the current, I= V/R


Here R = 5 ohm


V= 10V


I = 10/5 = 2 amp


(c) Potential difference across 2ohm resistor V= I x R1= 2×2=4V


(d) Potential difference across 3ohm resistor V= I x R2= 2×3=6V



Question 16.

In the circuit given below:



(a) What is the combined resistance?

(b) What is the p.d. across the combined resistance?

(c) What is the p.d. across the 3 Ω resistor?

(d) What is the current in the 3Ω resistor?

(e) What is the current in the 6 Ω resistor?


Answer:

A

The resistors of 6 ohm and 3 ohm are connected in parallel. Therefore, their combined resistance can be calculated as




R= 2 ohms


(b) The p.d. across the combined resistance, V= IR


Here I = 6A and combined resistance = 2 ohms.


So, V= 6 x 2 = 12 V


(c) In parallel connection potential difference remains constant, so Potential difference across 3ohm resistor =12 V


(d) The current in the 3Ω resistor


I= V/R1


I= 12/3 =4A


(e) Current flowing through the 6 ohm resistor = V/R2 = 12/6 = 2 A



Question 17.

A 5 V battery is connected to two 20 Ω resistors which are joined together in series.

(a) Draw a circuit diagram to represent this. Add an arrow to indicate the direction of conventional current flow in the circuit.

(b) What is the effective resistance of the two resistors?

(c) Calculate the current that flows from the battery.

(d) What is the p.d. across each resistor?


Answer:

A


(b) Since two resistors of 20 ohm are connected in series, the effective resistance will be


R= R1 +R2


Here, R1 = 20 ohm


R2 = 20 ohm


Therefore the effective resistance of the two resistors = 20+20 =40 ohm


(c) Current flowing through the circuit = I = V/R = 5/40 = 0.125 amps


(d) p.d. across each resistance = I x R = 0.125 x 20 = 2.5 V



Question 18.

The figure given below shows an electric circuit in which current flows from a 6V battery through two resistors.



(a) Are the resistors connected in series with each other or in parallel?

(b) For each resistor, state the p.d. across it.

(c) The current flowing from the battery is shared between the two resistors. Which resistor will have bigger share of the current

(d) Calculate the effective resistance of the two resistors.

(e) Calculate the current that flows from the battery.


Answer:

(a) In the given figure resistors are connected in parallel.

(b) In a parallel arrangement, the voltage remains the same across each resistor i.e 6V


(c) Due to lower resistance 2 ohm have bigger share of current.


(d) The resistors of 2 ohm and 3 ohm are connected in parallel. Therefore, their combined resistance can be calculated as




R= 1.2 ohm


(e) Current flowing through battery, I=V/R=6/1.2=5amps



Question 19.

A 4Ω coil and a 2 Ω coil are connected in parallel. What is their combined resistance? A total current of 3A passes through the coils. What current passes through the 2Ω coil?


Answer:

The coils of the resistors 4 ohm and 2 ohm are connected in parallel. Therefore



R=4/3 ohm


Total current =3 A (given)


Potential difference V= 3×4/3=4V


Now current through 2Ω coil= V/2 =4/2 = 2A




Long Answer Type Questions-pg-41
Question 1.

With the help of a circuit diagram, deduce the equivalent resistance of two resistances connected in series.


Answer:


Let the current in the circuit be I amperes and the battery be of strength V volts. Let the combined resistance of the three resistors be R ohms.


Therefore, according to Ohm's law, we have


V=IR ---- (i)


We know that when resistors are connected in series, the current with a battery of V volts.


V1=I×R1 ------- (ii)


V2 =I×R2 ------ (iii)


Let the potential difference across R1 is V1 and the potential difference across R2 is V2


V=V1+V2--------- (iv)


From equation (i),(ii), (iii) and (iv) we get.


IR= I×R1+ I×R2


R= R1+ R2



Question 2.

Two resistances are connected in series as shown in the diagram:



(i) What is the current through the 5 ohm resistance?

(ii) What is the current through R?

(iii) What is the value of R?

(iv) What is the value of V?


Answer:

(i) Current through 5 ohm resistor

I= V/R


I = 10/5 = 2A


(ii) Two resistances are connected in series. So same current flow through the circuit,


So current flowing through R =2A


(iii) As we know that according to ohm's law


V=IR


So R= V/I = 6/2 = 3 ohm


(iv) Total V in the given circuit = V1+V2 = 10+6= 16 V


Question 3.

With the help of a diagram, derive the formula for the resultant resistance of three resistors connected in series.


Answer:


Let the current be I amperes and the battery be of strength V volts. Let the combined resistance of the three resistors be R ohms.


Therefore, according to Ohm's law, we have


V= IR -----(i)


We know that when the resistors are connected in series, the current is the same in all the resistors. Therefore


V= V1 +V2+ V3 ------------------ (ii)


Let the current flowing through the whole circuit is I, and the equivalent resistance be R.


According to ohm's law


V= IR


V1= IR1 -------- (iii)


V2= IR2-------- (iv)


V3= IR3 ------ (v)


By using eq (i),(ii),(iii) ,(iv) and (v) we get


IR= IR1+ IR2+ IR3


R= R1+ R2+ R3



Question 4.

For the circuit shown in the diagram given below



Calculate:

(i) the value of current through each resistor.

(ii) the total current in the circuit.

(iii) the total effective resistance of the circuit.


Answer:

(i) Here R1= 5 ohm, R2= 10 ohm and R3= 30 ohm

The value of current through each resistor is different as they are arranged in parallel connection.


So the current through 5 ohm resistor, I1 = V/R = 6/5 = 1.2A


So the current through 10 ohm resistor, I2 = V/R = 6/10 = 0.6A


So the current through 30 ohm resistor, I3 = V/R = 6/30 = 0.2A


(ii) The total current in the circuit = I= I1+ I2+ I3


1.2A + 0.6A + 0.2A = 2A


(iii) Total resistance in the circuit = R, As the circuit is in parallel connection, so




R= 3 ohm



Question 5.

With the help of a circuit diagram, obtain the relation for the equivalent resistance of two resistances connected in parallel.


Answer:


Let the individual resistance of the two resistors be R1 and R2 and their combined resistance be R. Let the total current flowing in the circuit be I and strength of the battery be V volts. Then, from Ohm's law, we have: V= IR... (1)


We know that when resistors are connected in parallel, the potential drop across each resistance is the same. Therefore: =I=I1 + I2 = V/R1 + V/R2


I = V/(1 /R1 + 1/R2) ... (2)


From the equations (1) and (2) we have:


1/R = 1 /R1 + 1/R2



Question 6.

In the circuit diagram shown below, find:

(i) Total resistance.

(ii) Current shown by the ammeter A



Answer:

(i) Let the total resistance of the circuit =R

As the connections are parallel. So the total resistance will be


1/R=1/R1+1/R2


R2=3+2=5ohms


R1=5ohms


1/R=1/5+1/5


1/R=2/5


R=2.5ohms


(ii) Current shown by the ammeter A will be equal to the current following in the circuit.


I=V/R=4/(2.5)


=1.6amps



Question 7.

Explain with the help of a labelled circuit diagram, how you will find the resistance of a combination of three resistors of resistances R1, R2 and R3 joined in parallel.


Answer:


Let the resistance of the three resistors be R1, R2 and R3, respectively. Let their combined resistance be R.


Let the total current flowing in the circuit be I and the strength of the battery be V. Then from Ohm's law, we have: V= IR------- (1)


We know that when the resistors are connected in parallel, the potential drop across each resistance is the same. Therefore:


I = I1+I2+I3


I = V/R1+ V/R2+ V/R3


1/R=1/R1+1/R2+ 1/R3


If two resistances are connected in parallel, then the resultant resistance will be


1/R=1/R1+1/R2+ 1/R3



Question 8.

In the diagram shown below, the cell and the ammeter both have negligible resistance. The resistors are identical



With the switch K open, the ammeter reads 0.6 A. What will the ammeter reading when the switch is closed?


Answer:

When switch is open, the upper two resistances are connected in parallel in the circuit.

Effective resistance is 1/Req=1/R+1/R=2/R


Req = R/2


So the current=I=V/(R/2)=0.6A (given)


V/R = 0.3 A


When the switch closes, the third resistance also comes in the circuit.


The effective resistance of the circuit becomes R/3


Hence, Current I = V/(R/3) = 3 (V/R) = 3 x 0.3 = 0.9 A




Multiple Choice Questions (mcqs)-pg-42
Question 1.

The figure given below shows three resistors



Their combined resistance is:
A.

B. 14 Ω

C.

D.


Answer:

Here two resistor 6 ohm and 2 ohm are connected in parallel




Now R ohm and 6 ohm are in series


So the combined resistance


Question 2.

If two resistors of 35 Ω and 15 Ω are joined together in series and then placed in parallel with a 40 Ω resistor, the effective resistance of the combination is :
A. 0.1 Ω

B. 10 Ω

C. 20 Ω

D. 40 Ω


Answer:

When the resistors are in series, then the combined resistance = 35 Ω+15 Ω = 50Ω

Now 50Ω and 40 Ω are connected in parallel, so the combined resistance




Question 3.

The diagram below shows part of a circuit:



If this arrangement of three resistors was to be placed by a single resistor, its resistance should be :
A. 9 Ω

B. 4 Ω

C. 6 Ω

D. 18 Ω


Answer:

the two 6 Ω resistors are in parallel. Hence, the Equivalent Resistor =

Now, 3 Ω and 6 Ω resistors are in series. Hence the Total Equivalent Resistance = 3 + 6 = 9 Ω


Question 4.

In the circuit shown below :



The potential difference across the 3 Ω resistor is:
A.

B.

C. 1 V

D. 2V


Answer:

Here all the resistance are connected in series. Therefore net resistance = 2Ω+3Ω+1Ω =6Ω

So current in the circuit =


Potential across 3Ω resistor


Question 5.

A battery and three lamps are connected as shown:



Which of the following statements about the currents at X, Y and Z is correct?
A. The current at Z is greater than that at Y.

B. The current at Y is greater than that at Z.

C. The current at X equal the current at Y.

D. The current at X equals the current at Z.


Answer:

The current at Y is greater than that at Z due to the arrangement of the circuit.


Question 6.

V1, V2 and V3 are the p.d. across the 1 Ω, 2 Ω and 3 Ω resistors in the following diagram, and the current is 5A.



Which one of the columns (a) to (d) shows the correct values of V1, V2 and V3 measured in volts?
A. V1-1.0, V2-2.0, V3-3.0

B. V1-5.0, V2-10.0, V3-15.0

C. V1-5.0, V2-2.5, V3-1.6

D. V1-4.0, V2-3.0, V3-2.0


Answer:

As V = IR


Question 7.

A wire of resistance R1 is cut into five equal pieces. These five pieces of wire are then connected in parallel. If the resultant resistance of this combination be R2, then the ratio is:
A.

B.

C. 5

D. 25


Answer:

When the wire is cut into 5 pieces, resistance of each part is R/5

So, equivalent resistance of the 5 pieces when connected in parallel is



R1 = R/25


So required ratio R: R1=25:1



Questions Based On High Order Thinking Skills (hots)-pg-43
Question 1.

Show with the help of diagrams, how you would connect three resistors each of resistance 6 Ω so that the combination has resistance of (i) 9 Ω (ii) 4 Ω


Answer:

(i)


To obtain 9Ω resistance two resistance are connected in parallel,


Let the resultant resistance for parallel circuit=R


1/R=1/6+1/6


1/R=2/6


R=3


Effective resistance=6+3 = 9ohms


(ii) To obtain 4Ω resistance all resistance are connected in parallel



So the effective resistance will be R for each circuit



Or R= 2 Ω


Total resistance = 2 Ω+2 Ω=4Ω



Question 2.

To resistances when connected in parallel give resultant value of 2 ohm; when connected in series the value becomes 9 ohm. Calculate the value of each resistance.


Answer:



Question 3.

A resistor of 8 ohm. Calculated in parallel with another resistor X. The resultant resistance of the combination is 4.8 ohms. What is the value of the resistor X?


Answer:

Parallel combinations of the resistors will be written as

1/x + 1/8 = 1/4.8


On solving the above relation,


x = 12 ohms



Question 4.

You are given three resistances of 1, 2 and 3 ohms. Show by diagrams, how with the help of these resistances you can get :

(i) 6 Ω (ii) C (iii) 1.5 Ω


Answer:

(i) 6 ohm can be obtained by connecting the 1,2 and 3 ohm in series.


(ii) 6/11 ohm can be obtained by connecting the 1,2 and 3 ohm in parallel connection.



(iii) 1.5 ohm can be obtained by connecting 1 and 2 ohm in series and then by connecting 3 ohm to the resultant resistance i.e 3 ohm in parallel connection




Question 5.

How will you connect three resistors of 2Ω, 3Ω and 5Ω respectively so as to obtain a resultant resistance of 2.5 Ω? Draw the diagram to show the arrangement.


Answer:

Resistor 2 and 3 ohm are connected in series, 5 ohm resistance is connected parallel to 2 and 3 ohm.

Thus the resultant resistance will be 2.5 ohm




Question 6.

How will you connect three resistors of resistance 2Ω, 3 Ω and 6 Ω to obtain a total resistance of: (a) 4Ω, and (b) 1 Ω?


Answer:

(a) For total Resistance to be 4

we will connect 2 ohm resistance in series with parallel combination of 3 and 6 ohm resistance. When 3 and 6 ohm are in parallel combination then 1/R=1/3+1/6


or R=2


Now this R connected in series with 2 ohm resistance will give equivalent resistance = 2+ 2 = 4 ohm.


For total Resistance to be 1 ohm


All the given resistance should be connected in parallel combination 1/R’= 1/2+1/3+1/6


Or R'=1 ohm



Question 7.

What is (a) highest, and (b) lowest, resistance which can be obtained by combining four resistors having the following resistances?

4Ω, 8Ω, 12Ω, 24Ω


Answer:

(a) To obtain the highest resistance we must connect the given resistances in series.

Highest resistance R = 4 + 8 + 12 + 24 = 48ohms


(b) To obtain the lowest resistance we must connect the given resistances in parallel.


1/R=1/4+1/8+1/12+1/24


On solving we get, R=2ohms



Question 8.

What is the resistance between A and B in the figure given below?



Answer:

According to the figure, 20 ohm, 10 ohm and 20 ohm are connected in series so the resultant resistance in the extreme side = 20+10+20 = 50 ohms

Now this 50 ohm is in parallel with 30 ohm, so the resultant resistance become



On solving we get R = 18.75 ohm


Now 18.75 ohm, 10 ohm and 10 ohm are connected in series. So the resultant resistance become


R= 18.75+10+10 = 38.75 ohm.



Question 9.

You are given one hundred 1 Ω resistors. What is the smallest and largest resistance you can make in the circuit using these?


Answer:

The greatest possible resistance is the series combination of the resistors.

In series combination of hundred 1 ohm resistors equivalent resistance will be 100 ohm


The least possible resistance is the parallel combination of the resistors.


When resistances are connected in parallel then


1/R =100


R= 1/100 =0.01 ohm



Question 10.

You are supplied with a number of 100 Ω resistors. How could you combine some of these resistors to make a 250 Ω resistor?


Answer:

To make equivalent resistance of 250 ohms one may connect:

1) 2 resistors in series Thus, the resultant would be:


100+100= 200 ohms


2) 2 resistors in parallel Thus, the resultant of 2 parallel combination would be:



3) Now connect the combination of the two (series and parallel) in series: Thus, we may obtain a net resistance of: 200+50 = 250 ohms



Question 11.

The resistors R1, R2, R3 and R4 in the figure given below are all equal in value.



What would you expect the voltmeters A, B and C to read assuming that the connecting wires in the circuit have negligible resistance?


Answer:



Question 12.

Four resistances of 16 ohms each are connected in parallel. Four such combinations are connected in series. What is the total resistance?


Answer:

When four 16 ohm resistances is connected in parallel combination then the equivalent resistance will be

1/R = 1/16 + 1/16 + 1/16 + 1/16 = 4/16


R = 4 ohm


When four such combinations are connected in series, than the total resistance = 4+4+4+4 = 16 ohm.



Question 13.

If the lamps are both the same in the figure given below and if A1 reads 0.50 A, what do A2, A3, A4 and A5 read?



Answer:

It is given that the two lamps are identical. So, they will have equal resistance.

The ammeter A1 reads 0.5 A current. This current will now get distributed equally in the two branches as resistances are equal.


Hence, the ammeters A2 and A4 will read 0.25 A each.


Now, A3 and A5 are connected in series with A2 and A4, so they will also read 0.25 A.




Very Short Answer Type Questions-pg-47
Question 1.

Are the lights in your house wired in series?


Answer:

No, When appliances are connected in a parallel arrangement, each of them can be put on and off independently. This is a feature that is essential in a house's wiring.



Question 2.

What happens to the other bulbs in a series circuit if one bulb blows off?


Answer:

When bulbs are connected in a series arrangement, if one of them can be put off all bulbs are switched off.



Question 3.

What happens to the other bulbs in a parallel circuit if one bulb blows off?


Answer:

When bulbs are connected in a parallel arrangement, each of them can be put on and off independently.



Question 4.

Which type of circuit, series of parallel, is preferred while connecting a large number of bulbs:

(a) for decorating a hotel building from outside?

(b) for lighting inside the rooms of the hotel?


Answer:

(a) For decorating a hotel building from outside we use series connection.

(b) For lighting inside the rooms of the hotel we use parallel connection.



Question 5.

Draw a circuit diagram to show two 4 V electric lamps can be lit brightly from two 2 V cells.


Answer:





Short Answer Type Questions-pg-47
Question 1.

Why is series arrangement not used for connecting domestic electrical appliances in a circuit?


Answer:

In series arrangement if one electrical appliance stops working due to some defect, then all other appliance also stop working as the whole circuit is broken.So we should not used series arrangement for connecting domestic electrical appliances in a circuit.



Question 2.

Give three reasons why different electrical appliances in a domestic circuit are connected in parallel.


Answer:

Due to the following advantages different electrical appliances in a domestic circuit are connected in parallel:

(i) Each electrical appliance gets the same voltage as that of the power supply line.


(ii) When appliances are connected in a parallel arrangement, each of them can be put on and off independently.


(iii) If one electrical appliance stops working due to some defect, then all other appliances keep working properly.



Question 3.

Ten bulbs are connected in a series circuit to a power supply line. Ten identical bulbs are connected in a parallel circuit to an identical power supply line.

(a) Which circuit would have the highest voltage across each bulb?

(b) In which circuit would the bulbs be brighter?

(c) In which circuit, if one bulb blows out, all others will stop glowing?

(d) Which circuit would have less current in it?


Answer:

(a) Parallel circuit would have the highest voltage across each bulb.

(b) In Parallel circuit the bulbs will be brighter.


(c) In series circuit, if one bulb blows out, all others will stop glowing


(d) Series circuit would have less current in it.



Question 4.

Consider the circuits given below:



(a) In which circuit are the lamps dimmest?

(b) In which circuit or circuits are the maps or equal brightness to the lamps in circuit (i)?

(c) Which circuit gives out the maximum light?


Answer:

(a) In circuit (ii) the lamps are dimmest as it is connected in series arrangement.

(b) In circuit (iii) the lamps are equal brightness in circuit (i) as both are connected in parallel.


(c) Circuit (ii) give the maximum light.



Question 5.

If you were going to connect two light bulbs to one battery, would you use a series or a parallel arrangement? Why? Which arrangement takes more current from the battery?


Answer:

If we were going to connect two light bulbs to one battery, we would use a parallel arrangement, so the current will flow through them equally. Series arrangement take more current from battery.




Long Answer Type Questions-pg-48
Question 1.

Which is the better way to connect light and other electric appliances in domestic wiring: series circuits or parallel circuits? why?


Answer:

When appliances are connected in a parallel arrangement, each of them can be put on and off independently. This is a feature that is essential in a house's wiring.



Question 2.

Christmas tree lamps are usually wired in series. What happens if one lamp breaks?


Answer:

In series circuit, if one bulb blows out, all others will stop glowing.



Question 3.

An electrician has wired a house in such a way that if a lamp gets fused in one room of the house, all the lamps in other rooms of the house stop working. What is the defect in the wiring?


Answer:

The circuit arrangement is in series, so if one bulb blows out, all others will stop glowing.



Question 4.

Draw a circuit diagram showing two electric lamps connected in parallel together with a cell and a switch that works both lamps. Mark an (A) on your diagram to show where an ammeter should be placed to measure the current.


Answer:





Multiple Choice Questions (mcqs)-pg-48
Question 1.

The lamps in a household circuit are connected in parallel because:
A. this way they require less current

B. if one lamp fails the others remain lit

C. this way they require less power

D. if one lamp fails the others also fail


Answer:

The lamps in a household circuit are connected in parallel because in a parallel arrangement, each of them can be put on and off independently.


Question 2.

Using the circuit given below, state which of the following statement is correct?


A. When S1and S2 are closed, lamps A and B are lit.

B. With S1open and S2 closed, A is lit and B is not lit.

C. With S2 open and S1 closed A and B are lit.

D. With S1 closed and S2 open, lamp A remains lit even if lamp B gets fused.


Answer:

When S2 open and S1 closed A and B are lit.



Questions Based On High Order Thinking Skills (hots)-pg-48
Question 1.

Draw a circuit diagram showing two lamps, one cell and a switch connected in series.


Answer:




Question 2.

How can you change the brightness of the lamps?


Answer:

When lamps are connected in parallel the brightness of the lamps increases.



Question 3.

Consider the circuit given below where A, B and C are three identical light bulbs of constant resistance.



(a) List the bulbs in order of increasing brightness.

(b) If C burns out, what will be the brightness of A now compared with before?

(c) If B burns out instead, what will be the brightness of A and C compared with before?


Answer:

(a) Here A and B are connected in series so the brightness of the A and B will same while C are connected in parallel so the brightness of the bulb C is maximum.

(b) Their will be no effect on A and B.


(c) A and B are connected in series so if B burns than A also not glow but their will be no effect on C, as C is connected in parallel.



Question 4.

How do you think the brightness of two lamps arranged in parallel compares with the brightness of two lamps arranged in series (both arrangements having one cell)?


Answer:

The brightness of two lamps which are arranged in parallel is maximum than the the brightness of two lamps arranged in series.



Question 5.

If current flows through two lamps arranged:

(a) in series,

(b) in parallel,

and the filament of one lamps breaks, what happens to the other lamp? Explain your answer


Answer:

(a) In series circuit, if one bulb blows out, all others will stop glowing.

(b) When lamps are connected in a parallel arrangement, each of them is independently.



Question 6.

The figure below shows a variable resistor in a dimmer switch.



How would you turn the switch to make the lights:

(a) brighter, and

(b) dimmer?

Explain your answer.


Answer:

(a) To make the light brighter we turn the switch towards right side.

(b) To make the light dimmer we turn the switch towards left side.




Long Answer Type Questions-pg-6
Question 1.

What do you understand by the term “potential difference”?


Answer:

The difference in electric potential between two points is known as potential difference. The potential difference between two points in an electric circuit is defined as the amount of work done in moving a unit charge from one point to the other point.



Question 2.

What is meant by saying that the potential difference between two points is 1 volt?


Answer:

One volt is defined as the work done by one joule to move a charge of one coulomb from one place to another.



Question 3.

What is the potential difference between the terminals of a battery if 250 joules of work is required to transfer 20 coulombs of charge from one terminal of battery to the other?


Answer:

As we know that


Here, work done =250 Joule


Charge move =20 C

So,


Question 4.

What is the voltmeter? How is a voltmeter connected in the circuit to measure the potential difference between two points? Explain with the help of a diagram.


Answer:

Voltmeter is an apparatus used to measure the potential difference or electric potential difference between two points in an electric circuit.

The voltmeter is always connected in parallel across the two points where the potential difference is to be measured.




Question 5.

State whether a voltmeter has a high resistance or a low resistance. Give reason for your answer.


Answer:

A voltage has a high resistance so that it takes a negligible current from the circuit. The term volt gives rise to the word voltage. Voltage is the other name for potential difference




Very Short Answer Type Questions-pg-58
Question 1.

State two factors on which the electrical energy consumed by an electrical appliance depends.


Answer:

Two factors on which the electrical energy consumed by an electrical appliance depends on.

1). Power rating of the appliance.


2). Time for which the appliance is used.



Question 2.

Which one has a higher electrical resistance: a 100-watt bulb or a 60 watt bulb?


Answer:

As we know that


P is inversely proportional to resistance, so 60 watt has a higher electrical resistance.



Question 3.

Name the commercial unit of electric energy.


Answer:

The commercial unit of electric energy is Kilowatt-hour.



Question 4.

An electric bulb is rated at 220 V, 100 W. What is its resistance?


Answer:

Here, V = 220 V, P = 100W

R = ?


We know that



Thus


R = V2/P = 2202/100 = 484 ohm



Question 5.

What is the SI unit of (i) electric energy, and (ii) electric power?


Answer:

(i) The SI unit of electric energy is joule.

(ii) The SI unit electric power is watt.



Question 6.

Name the quantity whose unit is (i) kilowatt, and (ii) kilowatt-hour.


Answer:

(i) Kilowatt is the S.I unit of Electric power.

(ii) kilowatt-hour is the S.I unit of Electric energy.



Question 7.

Which quantity has the unit of watt?


Answer:

Watt is the SI unit of Electric power.



Question 8.

What is the meaning of the symbol kWh? Which quantity does it represent?


Answer:

kWh is the commercial unit of electrical energy. It full form is kilowatt-hour.



Question 9.

If the potential difference between the end of a wire of fixed resistance is doubled, by how much does the electric power increase?


Answer:

As we know that


i.e. as power is directly proportional to the square of potential difference. So, the electric power becomes four times its previous value.




Question 10.

An electric lamp is labeled 12 V, 36 W. This indicates that it should be used with a 12 V supply. What other information does the label provide?


Answer:

The electric lamp consumes energy at the rate of 36 J/s.



Question 11.

What current will be taken by a 920 W appliance if the supply voltage is 230 V.


Answer:

Here P = 920W, V = 230V, I = ?

We know that


P = V x I


920 = 230 x I


I = 920/230 = 4amp




Short Answer Type Questions-pg-58
Question 1.

Define watt. Write down an equation linking watts, volts and amperes.


Answer:

One watt is defined as the amount of electrical energy consumed by electrical appliances at the rate of 1 joule per second.

1 watt = 1 volt x 1 ampere



Question 2.

Define watt-hour. How many joules are equal to 1 watt-hour?


Answer:

The amount of electrical energy consumed by an electrical appliance of 1 watt power for 1 hour is called watt-hour.

1 watt hour is equal to 3600 joules



Question 3.

How much energy is consumed when a current of 5 amperes flows through the filament (or element) of a heater having resistance of 100 ohms for two hours? Express it in joules.


Answer:

Here, current (I) = 5amp, Resistance(R) = 100 ohms and time = 2h

As we know that


Electric energy consumed = P x t = I2Rt


= 52 x 100 x 2


= 5000 Wh


= 5 kwh


We know that


1kwh = 3.6 x 106 J


Therefore, 5kwh = 5 x 3.6 x 106 J = 18 x 106 J



Question 4.

An electric bulb is connected to a 220 V power supply line. If the bulb draws a current of 0.5 A, calculate the power of the bulb.


Answer:

Here, Potential difference (V) = 220V, Current (I) = 0.5amp, Power = ?

We know that


P = VI = 220 X 0.5


P = 110 watt



Question 5.

In which of the following cases ore electrical energy is consumed per hour?

(i) A current of 1 ampere passed through a resistance of 300 ohms.

(ii) A current of 2 amperes passed through a resistance of 100 ohms.


Answer:

(i) Here current (I) = 1 A, Resistance (R) = 300 ohm, time(t) = 1h

We know that


P = I2R = 12 x 300 = 300 W


E = P x t = 300 x 1 = 300 Wh


(ii) Here Current (I) = 2 A Resistance(R) = 100 ohm, time (t) = 1h


We know that


P = I2R = 22 x 100 = 400 W


E = P x t = 400 x 1 = 400 Wh


Hence, in case (ii), the electrical energy consumed per hour is more.



Question 6.

An electric kettle rated at 220 V, 2.2 kW, works for 3 hours. Find the energy consumed and the current drawn.


Answer:

Here potential difference(V) = 220V, Power (P) = 2.2kW = 2200W, time(t) = 3h

We know that


Electrical energy consumed = P × t = 2.2 ×3 = 6.6 kWh


We have, P = V x I


I = P/V


I = 2200/220


= 10amp



Question 7.

In a house two 60 W electric bulbs are lighted for 4 hours, and three 100 W bulbs for 5 hours every day. Calculate the electric energy consumed in 30 days.


Answer:

When two electric bulb of 60W are lighted for 4 hours then

Here, p1 = 60w


Number, n1 = 2


Time for daily use, t1 = 4h


Total electrical consumed everyday, E1 = n1×p1×t1


= 2×60×4 = 480


Energy consumed in 30 days = 30×0.48 = 14.4Kwh (As 1kW=1000W)


When three electric bulb of 100W are lighted for 5 hours then


Here, p2 = 100w


Number, n2 = 3


Time for daily use, t2 = 5h


Total electrical consumed every day, E1 = n2×p2×t2


= 3×100×5 = 1500


Energy consumed in 30 days = 30× 1.5 = 45Kwh (As 1kW= 1000 W)


Total electrical consumed in 30 days = 14.4Kwh + 45Kwh = 59.4 kWh


Question 8.

A bulb is rated as 250 V; 0.4 A Find its:

(i) power, and (ii) resistance.


Answer:

(i) We know that

V = 250V, I = 0.4amp


Power = VI = 250X0.4 = 100watt


(ii) We have,


V = 250V, I = 0.4amp


We know that


P = I2R


100 = 0.42XR


R = 625ohm



Question 9.

For a heater rated at 4 kW and 220 V, calculate:

(a) the current,

(b) the resistance of the heater,

(c) the energy consumed in 2 hours, and

(d) the cost if 1 kWh is priced at ₹ 4.60


Answer:

(a) Here, P = 4kw, V = 220v

I = ?


We know that


Power = VI = 220 × I


4000 = 220I


I = 4000/220= 18.18A


(b) Here, P = 4kw, V = 220v


R = ?


We know that,


P = I2R


P = (18.18)2×R


R = 4000/(18.18)2


R = 12.10 Ω


(c) Here, P = 4kw, time = 2hr


Energy consumed in two hour = P × t


= 4 × 2


= 8kw-hr


(d) If 1kwh = Rs 4.6


Here, Energy consumed in two hour = 8kwh


So, total cost = 8 x 4.6 = Rs 36.8


Question 10.

An electric motor take 5 amperes current from a 220 volts supply line. Calculate the power of the motor and electrical energy consumed by it in 2 hours.


Answer:

Here current (I) = 5amp, Potential difference(V) = 220 volt, time = 2h,

We have to find


Power (P) = ?


Energy (E) = ?


We know that


P = V×I


= 220×5


= 1100 watt


= 1.1 kW


Energy consumed, E = P × t


= 1.1 × 2


= 2.2 kWh



Question 11.

Which users more energy: a 250 W TV set in 1 hour or a 1200 W toaster in 10 minutes?


Answer:

TV set uses 0.25 kWh energy whereas toaster uses 1.20 kWh energy. So, toaster uses more energy.



Question 12.

Calculate the power used in the 2 Ω resistor in each of the following circuits:

(i) a 6V battery in series with 1 Ω and 2 Ω resistors.

(ii) a 4 V battery in parallel with 12 Ω and 2Ω resistors.


Answer:

(a) Here V = 6 volt, R1 = 1 ohm, R2 = 2 ohm

The connection is in series. So Equivalent resistance = R1+R2 = 1 + 2 = 3 ohm


We know that


Total current I = V/R = 6/3 = 2A


Current through R2 = I2 = I = 2A


Voltage across R2 = V2 = I2R2 = 2×2 = 4 ohm


Power used in R2 = I2V2 = 2×4 = 8W


(b) Here, V = 4 volt,R1 = 12 ohm, R2 = 2 ohm


We know that, as the connection is in parallel so voltage across R2 = V2 = V = 4V


Current across R2 = I2 = V2/R2 = 4/2 = 2A


As power = IV


Power used in R2 = I2V2 = 2×4 = 8W



Question 13.

Two lamps, one rated 40 W at 220 V and the other 60 W and 220 V, are connected in parallel to the electric supply at 220 V.
(a) Draw a circuit diagram to show the connections.

(b)Calculate the current drawn from the electric supply.

(b) Calculate the total energy consumed by the two lamps together when they operate for one hour.


Answer:

(a)

(b) As the connection is in parallel.

So voltage across both the bulbs is same and is equal to 220V.
Using the Formula P=VI,We can get I as I=P/V Where:-
P= Power
I=Current
V=Voltage/Potential Difference

Current through AB i.e 40W lamp = I1 = P1/V = 40/220 A

Current through CD i.e. 60W lamp = I2 = P2/V = 60/220 A

Total current drawn from the electric supply = 40/220 + 60/220 = 0.45 A

(c)We know that Energy= Power x Time

Total Power to two bulbs together=40W+60W=100W

According to the question both of the bulbs are being operated together at 1 hour

∴ Energy= 100W x 1hr
Energy= 100Whr

We know 1Whr= 3600 Joules
∴100Whr= 3600 X 100
=360000Joules
=360kJoules


Question 14.

An electric kettle connected to the 230 V mains supply draws a current of 10 A. Calculate:

(a) the power of the kettle.

(b) the energy transferred in 1 minute.


Answer:

(a) Here, V = 230V, I = 10amp

We know that, P = VI


P = 230X10


P = 2300watt = 2300 J/s


(b) The energy transferred in 1 minute. = P x t = 2300 J/s x 60s = 138000 J



Question 15.

A 2 kW heater, a 200 W TV and three 100 W lamps are all switched on from 6 p.m. to 10 p.m. What is the total cost at Rs. 5.50 per kWh?


Answer:

Here for heater we have, P = 2kW, t = 4h

So energy consumed by heater, E = P x t = 2x4 = 8kWh


Here for TV, we have, P = 200W = 0.2kW, time 10pm – 6pm = 4h


So energy consumed by TV, E = P x t = 0.2x4 = 0.8kWh


Here for TV, we have, P = 100W = 0.1kW, t = 4h, number of lamp(n) = 3


So energy consumed by three lamps, E = n x P x t = 3x0.1x4 = 1.2kWh


Total energy consumed by heater, TV and three lamps = 8+0.8+1.2 = 10kWh


Cost of 1kWh = Rs. 5.50


Cost of 10kWh = Rs. 5.50 x 10 = Rs. 55



Question 16.

What is the maximum power in kilowatts of the appliance that can be connected safely to a 13 A; 230 V mains socket?


Answer:

Here I = 13amp, V = 230V

As we know that


Power = VI


= 230 x 13


= 2990W


P = 2.99kW



Question 17.

An electric fan runs from the 230 V mains. The current flowing through it is 0.4 A. At what rate is electrical energy transferred by the fan?


Answer:

Here, V = 230V, I = 0.4amp

We know that power is the rate at which electric energy is transferred


P = V x I


= 230 x 0.4


= 92 W = 92 J/s




Long Answer Type Questions-pg-59
Question 1.

What is meant by “electric power”? Write the formula for electric power in terms of potential difference and current.


Answer:

The rate at which electrical energy is consumed, or the rate of doing electric work is known as electric power.

We know that


P = VI



Question 2.

The diagram below shows a circuit containing a lamp L, a voltmeter and an ammeter. The voltmeter reading is 3 V and the ammeter reading is 0.5 A.



(i) What is the resistance of the lamp?

(ii) What is the power of the lamp?


Answer:

(i) resistance = V/I = 3/0.5 = 6 Ω

(ii) P = VI = 3 × 0.5 = 1.5 Watt



Question 3.

Define kilowatt-hour. How many joules are the in one kilowatt-hour?


Answer:

The amount of electrical energy consumed when an electrical appliance having a power rating of 1 kilowatt is used for 1 hour is called One kilowatt hour.

1kWh = 3.6 x 106J



Question 4.

Calculate the cost of operating a heater of 500 W for 20 hours at the rate of ₹ 3.90 per unit.


Answer:

Here, Power of heater (P) = 500W = 0.5kW, Time(t) = 20hr

We know that


Energy consumed = P x t = 0.5 X 20


= 10kwh


Total cost = 10xcost per unit


Cost per unit = Rs. 3.9 per unit


Therefore, total cost = 10 x 3.9 = Rs 39




Multiple Choice Questions (mcqs)-pg-59
Question 1.

When an electric lamp is connected to 12 V battery, it draws a current of 0.5 A. The power of the lamp is :
A. 0.5 W

B. 6W

C. 12 W

D. 24W


Answer:

We know that

P = VI


P = 12× 0.5 = 6W


Question 2.

The unit for expressing electric power is:
A. volt

B. joule

C. coulomb

D. watt


Answer:

The unit for expressing electric power is watt


Question 3.

Which of the following is likely to be the correct wattage for an electric iron used in our homes?
A. 60 W

B. 250 W

C. 850 W

D. 2000 W


Answer:

The correct wattage for an electric iron used in our homes is 850 W.


Question 4.

An electric heater is rated at 2kW. Electrical energy costs ₹ 4 per kWh. What is the cost of using the heater for 3 hours?
A. ₹ 12

B. ₹ 24

C. ₹ 36

D. ₹ 48


Answer:

Energy consumed = P x t = 2 X 3 = 6kWh

Total cost = 6 x cost per unit


Cost per unit = Rs. 4 per kWh


Therefore, total cost = 6 x 4 = Rs 24


Question 5.

The SI unit of energy is:
A. joule

B. coulomb

C. watt

D. ohm-metre


Answer:

The SI unit of energy is joule.


Question 6.

The commercial unit of energy is:
A. watt

B. watt-hour

C. kilowatt-hour

D. kilo-joule


Answer:

The commercial unit of energy is kilowatt-hour.


Question 7.

How much energy does a 100 W electric bulb transfer in 1 minute?
A. 100 J

B. 600 J

C. 3600 J

D. 6000 J


Answer:

Energy consumed = P x t = 100 ×60 = 6000 joule

As in 1 minute = 60 second.


Question 8.

An electric kettle for use on a 230 V supply is rated at 3000 W. For safe working, the cable connected to it should be able to carry at least:
A. 2 A

B. 5 A

C. 10 A

D. 15 A


Answer:

As P = VI

I = P/V


Question 9.

How many joules of electrical energy are transferred per second by a 6V; 0.5 A lamp?
A. 30 J/s

B. 12 J/s

C. 0.83 J/s

D. 3 J/s


Answer:

As P = VI


Question 10.

At a given time, a house is supplied with 100 A at 220 V. How many 75 W, 220 V light bulbs could be switched on in the house at the same time (if they are all connected in parallel)?
A. 93

B. 193

C. 293

D. 393


Answer:

When they are connected in parallel 293 bulb will glow in the house at the same time.


Question 11.

If the potential difference between the ends of a fixed resistor is halved, the electric power will become:
A. double

B. half

C. four times

D. one-fourth


Answer:

If the potential difference between the ends of a fixed resistor is halved, the electric power will become one-fourth.



Questions Based On High Order Thinking Skills (hots)-pg-60
Question 1.

State whether an electric heater will consumer more electrical energy or less electrical energy per second when the length of its heating element is reduced. Given reasons for our answer.


Answer:

An electric heater will consume more electrical energy as power is inversely proportional to the resistance.



Question 2.

The table below shows the current in three different electrical appliances when connected to the 240 V mains supply:


(a) Which appliance has the greatest electrical resistance? How does the data show this?

(b) The lamp is connected to the mains supply by using a thin, twin-cored cable consisting of live and neutral wires. State two reasons why this cable should not be used for connecting the kettle to the mains supply.

(c) Calculate the power rating of the kettle when it is operated from the 240 V mains supply.

(d) A man takes the kettle abroad where the mains supply is 120 V. What is the current in the kettle when it is operated from the 120 V supply?


Answer:

(a) Lamp has the greatest electrical resistance as resistance oppose the flow of current, and the lamp has least amount of current.

(b) According to the given data large amount of current is needed by kettle, so it must be connected to the earthing.


(c) As we know that


P = VI


V = 240V, I = 8.5A


P = 240X8.5 = 2040 W = 2.04 kW


(d) When kettle is connected to 240 V supply, then the P = 2040 W


R = V2/P =


R = 28.23 ohm


Now, when V = 120V, R = 28.23 ohm


I = V/R = 120/28.23 = 4.25A



Question 3.

A boy noted the readings on his home’s electricity meter on Sunday at 8 AM and again on Monday at 8 AM (see Figures below).



(a) What was the meter reading on Sunday ?

(b) What was the meter reading on Monday?

(c) How many units of electricity have been used ?

(d) If the rate is Rs. 5 per unit, what is the cost of electricity used during this time?


Answer:

(a) The meter reading on Sunday is 42919.

(b) The meter reading on Monday is 42935.


(c) Units of electricity used = 42935- 42919 = 16 units.


(d) The cost of 1 unit is Rs 5. So the cost of 16 units = 16×5 = 80 rupees



Question 4.

An electric bulb is rated as 10 W, 220V. How many of these bulbs can be connected in parallel across the two wires of 220 V supply line if the maximum current which can be drawn in 5A ?


Answer:

Here it is given P = 10W, V = 220V, I = 5A

As we know that


P = VI


= 220X5


P = 1100W


Power of one bulb = 10W


So the total no. of bulbs that can be connected = 1100/10 = 110



Question 5.

Two exactly similar electric lamps are arranged (i) in parallel, and (ii) in series. If the parallel and series combination of lamps are connected to 220 V supply line one by one, what will be the ratio of electric power consumed by them?


Answer:

As we know that

Electric power consumed = V2/R


Here V is given we have to find the value of R


So when the connection is in parallel


The equivalent resistance is



Electric power consumed in parallel connection



When the connection is in Series


The equivalent resistance = R+R = 2R


Electric power consumed in series connection



The ratio of electric power consumed by them





Very Short Answer Type Questions-pg-66
Question 1.

How does the heat H produced by a current passing through a fixed resistance wire depend on the magnitude of current I?


Answer:

Heat produced, H = I2Rt

As heat produced is directly proportional to the square of current.



Question 2.

If the current passing through a conductor is doubled, what will be the change in heat produced?


Answer:

Heat produced, H = I2Rt

So, If the current passing through a conductor is doubled then heat produced becomes four times.



Question 3.

Name two effects produced by electric current.


Answer:

The two effects produced by electric current are:

(a) Heating effect of current


(b) Magnetic effect of current



Question 4.

Which effect of current is utilized in an electric light bulb?


Answer:

Heating effect of current is utilized in an electric light bulb.



Question 5.

Which effect of current is utilized in the working of an electric fuse?


Answer:

Heating effect of current is utilized in the working of an electric fuse.



Question 6.

Name two devices which work on the heating effect of electric current.


Answer:

The two devices which work on the heating effect of electric current are Electric heater and electric fuse.



Question 7.

Name two gases which are filled in filament type electric light bulbs.


Answer:

Argon and nitrogen are filled in filament type electric light bulbs,



Question 8.

Explain why, filament type electric bulbs are not power efficient.


Answer:

Due to the heating effect of current lots of electric power is consumed by the filament of a bulb



Question 9.

Why does the connecting cord of an electric heater not glow hot while the heating element does ?


Answer:

Due to the low resistance of copper the connecting cord of the heater does not glow because negligible heat is produced in it by passing current but the heating element made of nichrome glows because it becomes red-hot due to the large amount of heat produced on passing current (because of its high resistance).




Short Answer Type Questions-pg-66
Question 1.

Write down the formula for the heat produced when a current I is passed through a resistor R for time t.


Answer:

The heat produced when a current I is passed through a resistor R for time t will be

Heat produced, H = I2Rt



Question 2.

An electric iron of resistance 20 ohms draws a current of 5 amperes. Calculate the heat produced in 30 seconds.


Answer:

Here,

R = 20ohm, I = 5amp, t = 30s


We know that H = I2Rt


H = 52x20x30


H = 15000 J



Question 3.

State three factors on which the heat produced by an electric current depends. How does it depend on these factors?


Answer:

Heat produced by an electric current is given as H = I2Rt, that means

(i) Heat produced is directly proportional to square of current.


(ii) Heat produced is directly proportional to resistance.


(iii) Heat produced is directly proportional to the time for which current flows.



Question 4.

State and explain Joule's law of heating.


Answer:

Joule's law of heating states that when a current of I amperes flows in a wire of resistance R ohms for time t seconds then the heat produced in joules and is given by H = I2Rt

The law implies that heat produced in a resistor is (i) directly proportional to the square of current for a given resistance, (ii) directly proportional to resistance to resistance for a given current and (iii) directly proportional to the time for which current flows through the resistors.



Question 5.

A resistance of 40 ohms and one of 60 ohms are arranged in series across 220 volt supply. Find the heat in joules produced by this combination of resistances in half a minute.


Answer:

Here it is given that R1 = 40 ohms, R2 = 60 ohms, V = 220V, t = 30 sec

Here R1 and R2 are arranged in series connection.


So the equivalent resistance = R = 40+60 = 100 ohms


According to Ohm's law, we know that


V = IR


I = V/R


I = 220/100 = 2.2amp


As we know that heat produced in joules is given by H = I2Rt


Substituting the values of I, R and t in eq. H = I2RT


H = (2.2)2 X 100 X 30


H = 14520 J



Question 6.

Why is an electric light bulb not filled with air? Explain why argon or nitrogen is filled in an electric bulb.


Answer:

As the filament of the bulb is made from tungsten, so when we fill air inside the electric bulb this tungsten filament will burn quickly. So we fill unreactive gas like argon or nitrogen in the electric bulb to prolong the life of filament.



Question 7.

Explain why, tungsten is used for making the filaments of electric bulbs.


Answer:

Due to its very high melting point of tungsten it is used for making the filaments of electric bulbs



Question 8.

Explain why, the current that makes the heater element very hot, only slightly warms the connecting wires leading to the heater.


Answer:

Due to the low resistance of the connecting wires of the heater negligible heat is produced in them by passing current so it is slightly warm.



Question 9.

When a current of 4.0 A passes through a certain resistor for 10 minutes, 2.88 x 104 J of heat are produced. Calculate:

(a) the power of the resistor.

(b) the voltage across the resistor.


Answer:

(a), Here it is given,

I = 4 amp,


t = 10 min = 10 x 60 = 600sec, H = 2.88x104J


We Know that according to Joule's law of heating


H = I2RT


28800 = 42xRx600


R = 3ohms


We know that


P = I2xR


= 42x3


P = 48W


(b) Here we have to find the potential difference, V = ?


We know that


V = IR


V = 4x3


V = 12V



Question 10.

A heating coil has a resistance of 200 Ω. At what rate will heat be produced in it when a current of 2.5 A flows through it ?


Answer:

Here Resistance of the coil(R) = 200 ohms, Current (I) = 2.5 amp, time(t) = 1 sec

We know that according to Joule's law of heating


H = I2RT


H = (2.5)2×200×1


H = 1250 J/s



Question 11.

An electric heater of resistance 8 Ω takes a current of 15 A from the mains supply line. Calculate the rate at which heat is developed in the heater.


Answer:

Here Resistance (R) = 8 ohms, Current(I) = 15 amp, time (t) = 1 sec

We know that according to Joule's law of heating


H = I2RT


H = 152X8X1


H = 1800J/s



Question 12.

A resistance of 25 Ω is connected to a 12 V battery. Calculate the heat energy in joules generated per minute.


Answer:

Here R = 25ohms, V = 12V, H = ?, t = 60sec

According to Ohm’s law


V = IR


12 = 25XI


I = 0.48amp


We know that according to Joule's law of heating


H = I2RT


H = 0.482X25X60


H = 345.6J



Question 13.

100 joules of heat is produced per second in a 4 ohm resistor. What is the potential difference across the resistor?


Answer:

Here, it is given H = 100J, t = 1sec, R = 4ohms,

We know that according to Joule's law of heating


H = I2RT


100 = I2×4×1


100/4 = I2


I = 5amp


Again we know that


V = IR


V = 5X4


= 20V



Question 14.

Derive the expression for the heat produced due to a current 'I' flowing for a time interval 't' through a resistor 'R' having a potential difference 'V' across its ends. With which name is this relation known?


Answer:

All materials offer resistance to the flow of current through them. So some external energy is required to make the current flow. This energy is provided by the battery. Some of this energy gets dissipated as heat energy, so the resistor becomes hot.

Work done in carrying a charge Q through a potential difference V is given as


Also, Q = I t


Using Ohm's law, V = I R


W = I2 Rt


This work done in carrying the charge through the wire appears as the heat produced.


i.e. H = V I t = I2 Rt.


this energy is dissipated as heat energy.


This law is called Joule law of heating effect.



Question 15.

How much heat will an instrument of 12 W produce in one minute if it is connected to a battery of 12 V?


Answer:

Here, it is given that P = 12W, V = 12V, t = 60sec

We know that


P = VI


I = P/V = 12/12 = 1A


Again, we know that


V = IR


R = V/I = 12/1 = 12ohm


So according to the Joule’s heating law, we know that


H = I2Rt


H = 12x12x60


H = 720J



Question 16.

The current passing through a room heater has been halved. What will happen to the heat produced by it?


Answer:

Joule heating is given by

H = i2Rt


H is the heat produced, i is the current through the wire of resistance R for time t. If i is halved i.e i' = i/2


H become 1/4 th



Question 17.

What is meant by the heating effect of current? Give two applications of the heating effect of current.


Answer:

When electric current is supplied to a purely resistive conductor, the energy of electric current is dissipated entirely in the form of heat and as a result, resistor gets heated. The heating of resistor because of dissipation of electrical energy is commonly known as Heating Effect of Electric Current. Some examples are as follows:

When electric energy is supplied to an electric bulb, the filament gets heated because of which it gives light. The heating of electric bulb happens because of heating effect of electric current.


When an electric iron is connected to an electric circuit, the element of electric iron gets heated because of dissipation of electric energy, which heats the electric iron. The element of electric iron is a purely resistive conductor. This happens because of heating effect of electric current.



Question 18.

Name the material which is used for making the filaments of an electric bulb.


Answer:

Pure tungsten has some amazing properties including the highest melting point (3695 K), lowest vapor pressure, and greatest tensile strength out of all the metals. Because of these properties it is the most commonly used material for light bulb filaments.




Multiple Choice Questions (mcqs)-pg-66
Question 1.

The heat produced by passing an electric current through a fixed resistor is proportional to the square of:
A. magnitude of resistance of the resistor

B. temperature of the resistor

C. magnitude of current

D. time for which current is passed


Answer:

The heat produced by passing an electric current through a fixed resistor is proportional to the square of magnitude of current.

As we know that, H = V I t = I2 Rt.


Question 2.

The current passing through an electric kettle has been doubled. The heat produced will become:
A. half

B. double

C. four times

D. one-fourth


Answer:

The current passing through an electric kettle has been doubled. The heat produced will become four times.

As H = I 2 Rt


Question 3.

An electric fuse works on the:
A. chemical effect of current

B. magnetic effect of current

C. lighting effect of current

D. heating effect of current


Answer:

When the circuit current exceeds a specified value due to voltage fluctuations or short-circuiting, the fuse wire gets heated and melts.


Question 4.

The elements of electrical heating devices are usually made of:
A. tungsten

B. bronze

C. nichrome

D. argon


Answer:

An electric heating element is generally made from nichrome and can come in the shape of either a coil, ribbon or wire strip. When electricity is introduced into the heating element, its internal temperature increases and grows red hot as it begins to radiate heat outward


Question 5.

The heat produced in a wire of resistance ‘x’ when a current ‘y’ flows through it in time ‘z’ is given by :
A. x2 x y x z

B. x × z x y2

C. y x z2 x x

D. y x z x x


Answer:

As H = I 2 Rt

So, the heat produced in a wire of resistance ‘x’ when a current ‘y’ flows through it in time ‘z’ is given by x × z x y2


Question 6.

Which of the following characteristic is not suitable for a fuse wire?
A. thin and short

B. thick and short

C. low melting point

D. higher resistance than res of wiring


Answer:

In electronics and electrical engineering, a fuse is a type of low resistance resistor that acts as a sacrificial device to provide over current protection, of either the load or source circuit.


Question 7.

In a filament type light bulb, most of the electric power consumed appears as :
A. visible light

B. infra-red-rays

C. ultraviolet rays

D. fluorescent light


Answer:

In a filament type light bulb, most of the electric power consumed appears as infra-red-rays.


Question 8.

Which of the following is the most likely temperature of the filament of an electric light bulb when it is working on the normal 220 V supply line?
A. 500° C

B. 1500°C

C. 2500°C

D. 4500°C


Answer:

The automated mass manufacturing of incandescent filaments intended for operation at over 2500 K is a significant technical achievement


Question 9.

If the current flowing through a fixed resistor is halved, the heat produced in it will become:
A. double

B. one-half

C. one-fourth

D. four times


Answer:

If the current flowing through a fixed resistor is halved, the heat produced in it will become one-fourth

As H = I 2 Rt



Questions Based On High Order Thinking Skills (hots)-pg-67
Question 1.

The electrical resistivities of four materials P, Q, R and S are given below :

P - 6.84 × 10-8 Ω m

Q - 1.70 × 10-8 Ω m

R - 1.0 × 1015 Ω m

S - 11.0 × 10-7 Ω m

Which material will you use for making:

(a) heating element of electric iron

(b) connecting wires of electric iron

(c) covering of connecting wires? Give reason for your choice in each case.


Answer:

(a) The metals and alloy have very low resistivity in the range of 10-8 ohm meter to 10-6 ohm meter, so they are good conductor of electricity. While insulators have resistivity of the order of 1012 to 1017 ohm meter. Alloys have generally higher resistivity than its constituent metals and consider good for the heating elements. So S is used for heating element of electric iron.

(b) Q has very low resistivity of 1.7 X10-8ohm-m (it is actually copper). So it is used in connecting wires of electric iron.


(c) R has very high resistivity of 1.0X1015ohm-m (it is actually rubber). So it is used for covering of connecting wires.



Question 2.

How does the wire in the filament of a light bulb behave differently to the other wires in the circuit when the current flows ?


Answer:

Because of the property of low melting point it behaves differently from the other wires.



Question 3.

What property of the filament wire accounts for this difference?


Answer:

This is because of the high resistance offered by filament wire.



Question 4.

Two exactly similar heating resistances are connected (i) in series, and (ii) in parallel, in two different circuits, one by one. If the same current is passed through both the combinations, is more heat obtained per minute when they are connected in series or when they are connected in parallel? Give reason for your answer.


Answer:

Resultant resistance in a series combination is greater than any individual resistance, and resultant resistance in a parallel combination is smaller than any individual resistance.

Since His proportional to R; so more the resistance, more the heat. Therefore, the resistances must be connected in series to obtain more heat per minute.



Question 5.

An electric iron is connected to the mains power supply of 220 V. When the electric iron is adjusted at 'minimum heating' it consumes a power of 360 W but at 'maximum heating' it takes a power of 840 W. Calculate the current and resistance in each case.


Answer:

To find the heat produced by iron in the problem, we need to use an expression for produced power, which depends on the resistance of the iron and the current flowing through the iron.

Pmin = VI


360 = 220XI


I = 1.63amp


R = V/I


R = 220/1.63


R = 134.96ohms


In case of maximum heating we get


We know that


Pmax = VI


840 = 220XI


I = 3.81amp


R = V/I


R = 220/3.81


R = 57.74ohms



Question 6.

Which electric heating devices in your home do you think have resistors which control the flow of electricity?


Answer:

Electric fuse, Room heater, Electric iron are some of the device which have resistor which control the flow of electricity




Multiple Choice Questions (mcqs)-pg-6
Question 1.

The work done in moving a unit charge across two points in an electric circuit is a measure of:
A. Current

B. potential difference

C. resistance

D. power


Answer:

Potential difference between two points in a circuit is the work done in moving unit charge (i.e. one coulomb) from one point to the other.


Question 2.

The device used for measuring potential differences is known as:
A. potentiometer

B. ammeter

C. galvanometer

D. voltmeter


Answer:

The device used to measure the potential difference between two points in volts is known as voltmeter.


Question 3.

Which of the following units could be used to measure electric charge?
A. ampere

B. joule

C. volt

D. coulomb


Answer:

Coulomb is the SI unit of electric charge which is equal to the amount of charge transported by a current of one ampere in one second.


Question 4.

The unit for measuring potential difference is:
A. watt

B. ohm

C. volt

D. kWh


Answer:

Volt is the electrical unit of voltage or potential difference. One Volt is defined as energy consumption of one joule per electric charge of one coulomb.


Question 5.

One coulomb charge is equivalent to the charge contained in:
A. 2.6 X 1019 electrons

B. 6.2 X 1019 electrons

C. 2.65 X 1018 electrons

D. 6.25 X 1018 electrons


Answer:

One coulomb is the amount of charge that passes a point each second when the current is one amp. One coulomb is also equal to the charge contained in 6.24 x 1018 electrons.



Very Short Answer Type Questions-pg-11
Question 1.

By what name is the physical quantity coulomb /second called?


Answer:

SI unit of electric current is ampere (A). Ampere is the flow of electric charges through a surface at the rate of one coulomb per second.



Question 2.

What is the flow of charge called?


Answer:

The flow of electric charge is known as electric current. Electric current is carried by moving electrons through a conductor.



Question 3.

What actually travels through the wires when you switch on a light?


Answer:

When we switch on a light electrons or charge are travelled through the wire.



Question 4.

Which particles constitute the electric current in a metallic conductor?


Answer:

The electric current is constituted by electron in a metallic conductor.



Question 5.

In which direction does conventional current flow around a circuit?


Answer:

The direction of an electric current is by convention the direction in which a positive charge would move. Thus, the current in the external circuit is directed away from the positive terminal and toward the negative terminal of the battery.



Question 6.

In which direction do electrons flow?


Answer:

Electrons would actually move through the wires in the opposite direction i.e. from negative terminal to positive terminal.



Question 7.

Which of the following equation shows the correct relationship between electrical units?

1 A = 1 C/s or 1 C = 1 A/s


Answer:

If 1 coulomb of electric charge flows through a cross section for 1 second, it would be equal to 1 ampere.


Therefore; 1 A = 1 C/1 s



Question 8.

What is the unit of electric current?


Answer:

SI unit of electric current is ampere (A).



Question 9.

How many milli-amperes are there in 1 ampere?


Answer:

Small quantity of electric current is expressed in milliampere and microampere. Milliampere is written as mA and microampere as μA


1mA (milliampere)= 10−3 A



Question 10.

How many micro-amperes are there in 1 ampere?


Answer:

1μA(microampere)=10−6 A



Question 11.

Which of the two is connected in series: ammeter or voltmeter?


Answer:

If the voltmeter is connected in series then the potential diff between two consecutive points will be zero. Thus voltmeter is useless. Ammeter is used to measure the current flowing in a branch.



Question 12.

Compare how an ammeter and a voltmeter are connected in a circuit.


Answer:

If the voltmeter is connected in series then the potential diff between two consecutive points will be zero. Thus voltmeter is useless. Ammeter is used to measure the current flowing in a branch. If it is connected in parallel, then the current will be divided between different branches. So voltmeter is connected in parallel and ammeter is connected in series.



Question 13.

What do the following symbols mean in circuit diagrams?

(i)

(ii)


Answer:

(i) This symbol represents variable resistance or rheostat


(ii) This symbol represents a closed switch or a closed plug key.



Question 14.

If 20 C of charge pass a point in a circuit in 1 s, what current is flowing?


Answer:

The flow of electric charge is known as electric current. Here 20 C charge pass in a circuit in one second.




Question 15.

A current of 4 A flows around a circuit for 10 s. How much charge flows past a point in the circuit in this time?


Answer:

Here current I = 4 A and Time = 10 sec


So charge = Current × time


= 4 × 10 = 40 C



Question 16.

What is the current in a circuit if the charge passing each point is 20 C in 40 s ?


Answer:

Here charge =20 C and time = 40 sec




Question 17.

Fill in the following blanks with suitable words :

(a) A current is a flow of .................For this to happen there must be a ...........circuit.

(b) Current is measured in................ using an .................. placed in ............ in a circuit.


Answer:

(a) Electric current is carried by moving electrons through a conductor. Electric circuit is a continuous and closed path of electric current.


(b) Current is measured in ampere using an ammeter placed in series in a circuit.




Short Answer Type Questions-pg-11
Question 1.

Name a device which helps to maintain potential difference across a conductor (say, a bulb).


Answer:

A source of electricity such as cell, battery, power supply, etc. helps to maintain a potential difference across a conductor.



Question 2.

If a potential difference of 10 V causes a current of 2 A to flow for 1 minute, how much energy is transferred?


Answer:

As we know that


Q = I × t


Here I= 2 A and T = 1 minute = 60 sec


Thus Q = 120 C


Again as we know


V = W/Q


W = VQ


W = 10 × 120 = 1200 joule



Question 3.

What is an electric current? What makes an electric current flow in a wire?


Answer:

The flow of electric charge is known as electric current. Electric current is carried by moving electrons through a conductor. It is the potential difference between the ends of the wire which make the electric charge (or current) to flow in the wire.



Question 4.

Define the unit of electric current (or Define ampere).


Answer:

SI unit of electric current is ampere (A). Ampere is the flow of electric charges through a surface at the rate of one coulomb per second.



Question 5.

What is an ammeter? How is it connected in a circuit? Draw a diagram to illustrate your answer.


Answer:

An ammeter is an apparatus which is used to measure electric current in a circuit. An ammeter is always connected in series with the circuit in which the current is to be measured.




Question 6.

Write down the formula which relates electric charge, time and electric current.


Answer:

The equation below shows the relationship between charge, current and time: charge (coulomb, C) = current (ampere, A) × time


Q = I × t



Question 7.

A radio set draws a current of 0.36 A for 15 minutes. Calculate the amount of electric charge that flows through the circuit.


Answer:

Here I = 0.36 A


Time = 15 minute = 15 × 60 = 900 sec


So Charge = 0.36 × 900 = 324 C



Question 8.

Why should the resistance of :

(a) an ammeter be very small?

(b) a voltmeter be very large?


Answer:

(a) An ammeter has to measure to current flowing through the circuit. Resistance offers an obstruction to the current flow. So, if the resistance of an ammeter is large, the current measured by the ammeter will be quite less as compared to the actual amount of current flowing through the circuit which is undesirable. If ammeter has zero resistance, then it will give the exact value of current. But this is not practically possible because every material has some value of internal resistance which we can't control. For this reason, ammeter must have small resistance.


(b) If current is flowing through the voltmeter, then it is not all flowing through the load, and the potential difference across the load would change when the voltmeter is added and removed. This is unfavourable. Therefore, the voltmeter must have a very high resistance so that current doesn't flow through it.



Question 9.

Draw circuit symbols for (a) fixed resistance (b) variable resistance (c) a cell (d) a battery of three cells(e) an open switch (j) a closed switch.


Answer:




Question 10.

What is a circuit diagram? Draw the labelled diagram of an electric circuit comprising of a cell, a resistor, an ammeter, a voltmeter and a closed switch (or closed plug key). Which of the two has a large resistance: an ammeter or a voltmeter?


Answer:

Electric circuit is represented by drawing circuit diagrams. A diagram which indicates how different components in a circuit have been connected by using the electrical symbols for the components is called a circuit diagram.



Voltmeter is an apparatus used to measure the potential difference or electric potential difference between two points in an electric circuit. A voltmeter has a large resistance.



Question 11.

If the charge on an electron is 1.6 x I0-19 coulombs, how many electrons should pass through a conductor in 1 second to constitute 1-ampere current?


Answer:

We know that charge over 1 electron = 1.6 × 10-19 coulomb

Thus, 1.6 × 10-19 C of charge = 1 electron

Therefore, 1 C of charge


Question 12.

The p.d. across a lamp is 12 V. How many joules of electrical energy are changed into heat and light when:

(a) a charge of 1 C passes through it?

(b) a charge of 5 C passes through it?

(c) a current of 2 A flows through it for 10 s?


Answer:

As we know that


Work done = potential difference x charge


a) W= 12 ×1= 12 j


b) W= 12 ×5 = 60 j


c) W = 12 × 2 x 10 = 240 j



Question 13.

In 10 s, a charge of 25 C leaves a battery, and 200 J of energy are delivered to an outside circuit as a result.

(a) What is the p.d. across the battery?

(b) What current flows from the battery?


Answer:

(a) As we know that



(b)




Long Answer Type Questions-pg-12
Question 1.

Define electric current. What is the SI unit of electric current.


Answer:

The flow of electric charge is known as electric current. Electric current is carried by moving electrons through a conductor. SI unit of electric current is ampere (A).



Question 2.

One coulomb of charge flows through any cross-section of a conductor in 1 second. What is the current flowing through the conductor?


Answer:

One coulomb is nearly equal to 6 x 1018 electrons. SI unit of electric current is ampere (A). Ampere is the flow of electric charges through a surface at the rate of one coulomb per second. This means if 1 coulomb of electric charge flows through a cross section for 1 second, it would be equal to 1 ampere.



Question 3.

Which instrument is used to measure electric current? How should it be connected in a circuit?


Answer:

An ammeter is an apparatus which is used to measure electric current in a circuit. It should be connected in series with the circuit.



Question 4.

What is the conventional direction of the flow of electric current? How does it differ from the direction of flow of electrons?


Answer:

The direction of an electric current is by convention the direction in which a positive charge would move. Thus, the current in the external circuit is directed away from the positive terminal and toward the negative terminal of the battery. Electrons would actually move through the wires in the opposite direction i.e from negative terminal to positive terminal.



Question 5.

A flash of lightning carries 10 C of charge which flows for 0.01 s. What is the current? If the voltage is 10 MV, what is the energy?


Answer:

Here


Charge, Q = 10C


Voltage = 10 MV = 10 x 106V


Energy of the charge particle is given by


E = Q V


= 10 x 10 x 106


=108 J




Multiple Choice Questions (mcqs)-pg-12
Question 1.

The other name of potential difference is :
A. ampereage


B. wattage

C. voltage

D. potential energy


Answer:

Electric potential difference is known as voltage, which is equal to the work done per unit charge to move the charge between two points against static electric field. It is denoted by ‘V’.


Question 2.

Which statement/statements is/are correct?

1) An ammeter is connected in series in a circuit and a voltmeter is connected in parallel.

2) An ammeter has a high resistance.

3) A voltmeter has a low resistance.
A. 1, 2, 3


B. 1, 2

C. 2, 3

D. 1


Answer:

An ammeter has to measure to current flowing through the circuit. Resistance offers an obstruction to the current flow. So, if the resistance of an ammeter is large, the current measured by the ammeter will be quite less as compared to the actual amount of current flowing through the circuit which is undesirable. If ammeter has zero resistance, then it will give the exact value of current. But this is not practically possible because every material has some value of internal resistance which we can't control. For this reason, ammeter must have small resistance.


Question 3.

Which unit could be used to measure current?
A. Watt


B. Coulomb

C. Volt

D. Ampere


Answer:

SI unit of electric current is ampere (A).


Question 4.

If the current through a flood lamp is 5 A, what charge passes in 10 seconds?
A. 0.5 C


B. 2 C

C. 5 C

D. 50 C


Answer:

Charge = Current × time = 10 × 5 = 50


Question 5.

If the amount of electric charge passing through a conductor in 10 minutes is 300 C, the current flowing is :
A. 30 A


B. 0.3 A

C. 0.5 A

D. 5 A


Answer:

Charge = Current × time



Questions Based On High Order Thinking Skills (hots)-pg-12
Question 1.

A student made an electric circuit shown here to measure the current through two lamps.

A. Are the lamps in series or parallel?

B. The student has made a mistake in this circuit.

What is the mistake?

C. Draw a circuit diagram to show the correct way to connect the circuit.

Use the proper circuit symbols in your diagram.


Answer:

(a) The lamps are connected in series. In a series circuit, every device must function for the circuit to be complete.


(b) Here the students connected Ammeter in parallel with the lamps. An ammeter is always connected in series.


(c)



Question 2.

Draw a circuit diagram to show how 3 bulbs can be lit from a battery so that 2 bulbs are controlled by the same switch while the third bulb has its own switch.



Answer:

Two bulbs are connected in series, so that both bulbs are controlled by same switch and the third bulb is connected in parallel, so that it can be controlled by another switch.




Question 3.

An electric heater is connected to the 230 V mains supply. A current of 8A flows through the heater.

(a) How much charge flows around the circuit each second?

(b) How much energy is transferred to the heater each second?


Answer:

(a) Here Potential difference = 230 V


Current = 8A


Time = 1 Second


As we know that


The flow of electric charge per unit time is known as electric current.




So in each second the amount of charge flowing in the circuit is 8 C.


(b) As we know that the work done is equal to the amount of energy transferred in a circuit.


Electric potential difference is known as voltage, which is equal to the work done per unit charge to move the charge between two points against static electric field.


So,



So, Work done = 230 × 8 = 1840 J


It is also equal to the total energy transferred to the heater in each second



Question 4.

How many electrons are flowing per second past a point in a circuit in which there is a current of 5 amp ?


Answer:

Here


Q = I × t


As we know that


Q = ne


Ne = I × t